Barbri last 100 MBE practice
A young woman who attended a rock concert at a nightclub was injured when the band opened its performance with illegal fireworks that ignited foam insulation in the club's ceiling and walls. The young woman sued the radio station that sponsored the performance. The radio station has moved for summary judgment, claiming that it owed no duty to audience members. The evidence has established the following facts: The station advertised its sponsorship on the radio and in print, distributed free tickets to the concert, staffed the event with the station's interns to assist with crowd control, and provided a station disc jockey to serve as master of ceremonies. The master of ceremonies had the authority to stop or delay the performance at any time on the basis of any safety concern. The station knew or should have known that the band routinely used unlicensed, illegal fireworks in its performances. Should the court grant the radio station's motion for summary judgment? Press Enter or Space to submit the answerANo, because there is sufficient evidence of knowledge and control on the part of the station to impose on it a duty of care to audience members.BNo, because under respondeat superior, the radio station is vicariously liable for the negligent actions of the band.CYes, because it is the band and the nightclub owners who owed audience members a duty of care.DYes, because the conduct of the band in setting off illegal fireworks was criminal and was a superseding cause as a matter of law.
(A) is correct. If a defendant's conduct creates an unreasonable risk of injury to persons in the position of the plaintiff, the general duty of care extends from the defendant to the plaintiff. Here, based on all of the facts, the court could find that a duty of care extends from the radio station to the audience members. The station knew or should have known that the band routinely used unlicensed, illegal fireworks in its performances, and the station's disc jockey had the power to stop or delay the performance for safety reasons. Given the radio station's extensive control over the concert, its motion for summary judgment should be denied. (B) is incorrect. The doctrine of respondeat superior applies when an employee acting within the scope of employment commits a tortious act, making the employer vicariously liable. Here, the band was not employed by the radio station, so respondeat superior does not apply. (C) is incorrect. While the band and nightclub owners did owe the audience members a duty of care, the radio station also owed a duty based on the facts provided, as discussed above. (D) is incorrect. If the defendant's negligence created a foreseeable risk that a third person would commit a crime or intentional tort, the defendant's liability will not be cut off by the crime or tort. Here, the trier of fact could determine that the radio station's failure to make sure illegal fireworks were not used or to immediately stop the performance when the band used them constituted negligence and created a foreseeable risk that the band would set off the illegal fireworks and injure someone. Accordingly, the court cannot hold that the band's conduct was a superseding cause as a matter of law.
A defendant was charged with and tried for murder. At trial, the defendant testified that he had acted in self-defense. The trial court instructed the jury that the defendant had the burden of proving by a preponderance of the evidence that he had acted in self-defense. The defendant objected, arguing that this instruction violated the constitutional requirement that the prosecution prove the offense beyond a reasonable doubt. Did the trial court's instruction violate the defendant's federal constitutional rights? Press Enter or Space to submit the answerANo, because due process does not preclude requiring a defendant to prove self-defense by a preponderance of the evidence.BNo, because due process does not regulate the burdens of proof in criminal cases.CYes, because due process precludes placing any evidentiary burden on a criminal defendant.DYes, because due process precludes requiring a criminal defendant to bear the burden on an issue that would make an act lawful.
(A) is correct. In all criminal trials, the government has the burden to prove all the elements of a crime beyond a reasonable doubt. However, this rule does not preclude the state from imposing a burden on the defendant to prove an affirmative defense, such as self-defense. Therefore, (C) and (D) are incorrect. (B) is incorrect because it is an inaccurate statement of the law. As mentioned above, due process requires the government to prove all elements of a crime beyond reasonable doubt.
A police officer stopped a driver who had run a red light. Upon approaching the car, the officer noticed a strong odor of alcohol and immediately asked whether the driver had been drinking. The driver admitted having had several alcoholic drinks that evening. The driver, charged with driving while intoxicated, moved to suppress the officer's testimony regarding the driver's statement about his drinking. The driver argued that the officer had elicited the statement without providing the requisite Miranda warnings. The prosecutor has responded that the statement should be allowed in the prosecution's case-in-chief or, at a minimum, should be allowed as impeachment in the event the driver testifies and denies drinking. How should the court rule regarding the driver's statement admitting his drinking? Press Enter or Space to submit the answerAThe statement should be allowed, because although the driver was in custody, the officer's spontaneous utterance upon smelling alcohol did not rise to the level of interrogation.BThe statement should be allowed, because the driver was not in custody for Miranda purposes when the admission was made.CThe statement should be suppressed both in the prosecution's case-in-chief and as impeachment evidence, even if the driver testifies.DThe statement should be suppressed in the prosecution's case-in-chief, but it may be used as impeachment evidence if the driver testifies.
(B) is correct. As a general rule, to offset the coercive nature of custodial interrogation by a police officer and protect defendants' Fifth Amendment right against compelled self-incrimination, the Supreme Court has made Miranda warnings a prerequisite to the admissibility of confessions obtained during a custodial police interrogation. The officer's question here was an interrogation, as the term "interrogation" extends to any words or conduct used by an officer intentionally to elicit an incriminating response. Nevertheless, the driver's response here is admissible despite the lack of Miranda warnings because the driver was not in custody. Determining whether a person is in custody for Miranda purposes involves a two-step inquiry. The Court will first look to whether a reasonable person in the circumstances would feel free to terminate the encounter and leave. If not, the Court looks to the whether the relevant environment presents coercive pressures similar to those of a station house interrogation. The more a setting resembles a traditional arrest, the more likely the Court will find the person to be in custody. Here, the driver, no doubt, felt that he was not free to leave, having just been pulled over for a traffic violation. However, a traffic stop does not resemble a traditional arrest. Given their brief nature and the motorist's knowledge that he will soon be on his way, the Supreme Court has found traffic stops to be noncustodial. Therefore, a police officer can ask questions during a routine traffic stop without giving Mirandawarnings, and responses to those questions are admissible at trial in a case-in-chief and for impeachment purposes. Therefore, (A), (C), and (D) are incorrect.
A man and his neighbor owned homes on adjacent lots in a subdivision. The subdivision's recorded restrictions did not prohibit detached storage sheds, and several homeowners in the subdivision had placed such sheds in their backyards. Because the man and the neighbor thought the sheds were unsightly, they both agreed in writing not to place detached storage sheds in their respective yards. Their agreement was drafted in recordable form and stated that it was enforceable by and against all assignees, heirs, and successors. The agreement was promptly recorded. Three years later, the neighbor gave his home to his daughter. Shortly after moving into the home, the daughter learned of the restriction. She informed the man that she planned to put a detached storage shed in her backyard, claiming that the restriction was not enforceable against her. Does the man have the right to enjoin the neighbor's daughter from placing a detached storage shed in her yard? Press Enter or Space to submit the answerANo, because several homeowners in the subdivision have storage sheds in their yards.BNo, because there was no horizontal privity between the man and the neighbor.CYes, because the neighbor conveyed the home to the daughter by gift rather than by sale.DYes, because the restriction is binding on the daughter as a successor.
(D) is correct. A restrictive covenant can be enforced at law for damages (as a real covenant running with the land) or in equity for an injunction (equitable servitude). The requirements are different for each. Here the facts state that the man is seeking an injunction to prevent the neighbor's daughter from placing a shed in her yard. Thus, the man is trying to enforce the restrictive covenant as an equitable servitude. An enforceable equitable servitude requires that the covenanting parties intended the servitude to be enforceable against successors in interest, the successor in interest must have notice of the covenant, and the covenant must touch and concern the land. Here, the original parties clearly intended their successors to be bound because they stated in the agreement that it was enforceable by and against all assignees, heirs, and successors. The agreement was recorded. Thus, all successors would have constructive (record) notice of the covenant. For the benefit of a covenant to touch and concern the land, the promised performance must benefit the covenantee and their successors in their use and enjoyment of the benefited land. Here, the performance (not building a shed) benefited the man and his successors in their use and enjoyment of the land because they could enjoy their property without viewing sheds they consider unsightly. All requirements for an equitable servitude are met, and the main has a right to enjoin the neighbor's daughter. (A) is incorrect because the question concerns enforcing an agreement between two parties; it does not involve a common development scheme. Thus, what the other neighbors do is irrelevant. (B) is incorrect. Horizontal privity is required for a burden of a real covenant to run at law to successors in interest. It would have been required had the man been seeking damages rather than an injunction. Horizontal privity requires that the original covenanting parties share some interest in the land independent of the covenant (e.g., grantor-grantee). Here, the parties to the agreement were merely neighbors and had no legal relationship; thus there was no horizontal privity. As noted above, the man may still enforce the agreement as an equitable servitude. (C) is incorrect. The fact that the daughter took the property as a gift prevents her from being a bona fide purchaser for value ("BFP"). A BFP takes free of a restrictive covenant if they had no notice of it. Here, the daughter is not a purchaser and she had record notice of the covenant. Thus, she would take subject to the restriction if it were enforceable against her. So the question is whether the restriction is enforceable against the neighbor's successor in interest. The answer is yes.
Ten years ago, a couple bought a building and moved into its second-floor apartment with their teenage daughter. The couple operated a shoe store on the first floor of the building for many years. When the couple purchased the building, the area was predominantly rural and was zoned for nonresidential use. The municipality's zoning is cumulative. Five years ago, the municipality rezoned the area to single-family residential use. The daughter was not aware of this change, since she was away at college. Recently, the daughter inherited the building from her parents. The daughter immediately moved into the apartment and took over the operation of the shoe store on the first floor. The daughter has learned that a developer is planning to build a large residential community in the area surrounding her building. The daughter has asked her lawyer for advice regarding her ability to continue operating the shoe store. Should the lawyer advise the daughter that she can continue to operate her shoe store? Press Enter or Space to submit the answerANo, because the nonconforming use of the building terminated when the daughter's parents died.BNo, but the municipality must pay her reasonable compensation for her loss resulting from the change in zoning.CYes, because the shoe store is a nonconforming use.DYes, because the zoning is cumulative and the building is also used for single-family residential purposes.
(C) is correct. A cumulative zoning ordinance creates a hierarchy of uses of land, and land that is zoned for a particular use may be used for the stated purpose or for any higher use. A residential use is higher than a nonresidential use. Here, the building was in an area originally zoned for nonresidential use. The daughter and her parents used the property for a business and their residence. This was appropriate under the cumulative zoning ordinance as the family's uses met or exceeded the zoned use. Later, the area was rezoned for single-family residential use, which is a higher use than the shoe store. However, a use that exists at the time of passage of a zoning ordinance and that does not conform cannot be eliminated at once. Generally, the nonconforming use may continue indefinitely, but any change in the use must comply with the zoning ordinance. Because the shoe store existed at the time of the rezoning, the daughter may continue to operate the shoe store as a nonconforming use. (A) is incorrect. Generally, the nonconforming use may continue indefinitely. The key is the use of the property and not the ownership. The parents' deaths do not affect the zoning status of the building. Thus, the daughter may live in the building and operate the shoe store. (B) is incorrect. The zoning power is limited by the "no taking without just compensation" clause of the Fifth Amendment. Nevertheless, zoning is generally not a "taking" unless it amounts to a physical appropriation of the property or denies the owner of all economic use, which would not be the case here (the daughter could use the property as a single-family residence). Because the property is a nonconforming use, she will be allowed to continue that use. (D) is incorrect. A cumulative zoning ordinance creates a hierarchy of uses of land, and land that is zoned for a particular use may be used for the stated purpose or for any higher use. A single-family home is a higher use than an apartment building or a commercial use such as a shoe store. Because the shoe store is a lower use than a single-family residence, it would not be allowed in this area. Nor would the daughter's apartment above the store. The store and apartment are allowed only because they are a nonconforming use.
A man decided to give a cabin he owned to his daughter at his death. To accomplish this goal, he delivered to his attorney a deed that fully complied with the applicable statute of frauds and told his attorney to record the deed when he died unless he later gave the attorney instructions to the contrary. Three weeks after dropping off the deed, the man properly drafted and executed his own will, which left all of his real property to his son. One year later, the man died, and the attorney immediately recorded the deed. At the time of the man's death, the cabin was titled in his name and he owned no other real property. The daughter and the son now disagree as to who is entitled to ownership of the cabin. Other than the jurisdiction's statute of frauds and statute of wills, there are no applicable statutes. Who is entitled to ownership of the cabin? Press Enter or Space to submit the answerAThe daughter, because the attorney was, for gift-law purposes, a trustee for the daughter.BThe daughter, because the deed fully complied with the statute of frauds.CThe son, because the deed was not delivered to the daughter during the man's lifetime.DThe son, because the proper execution of the will revoked the earlier gift to the daughter.
(C) is correct. A deed is not effective to transfer an interest in realty unless it has been delivered. Delivery refers to the grantor's intent. There must be words or conduct showing the grantor intended that the deed have some present operative effect- that title pass immediately and irrevocably, even though the right of possession may be postponed. If the right of possession is to be postponed until the grantor's death, the deed may be held "testamentary" and therefore void (unless executed with testamentary formalities). However, most courts hold that if the grantor executes a deed and gives it to another with instructions to give it to the grantee upon the grantor's death, the grantor's intent was to presently convey a future interest to the grantee (either a remainder, with a life estate reserved in the grantor, or an executory interest), and so the gift is inter vivos, not testamentary. To make an effective delivery, the grantor must relinquish absolute and unconditional control. That was not the case here. The man gave the deed to the attorney to record when the man died "unless he later gave the attorney instructions to the contrary." This language shows that the man did not relinquish control. Thus, there was no delivery, and the property passed by the will to the son. (A) is incorrect. A trustee holds legal title to property and has an equitable duty to deal with it for the benefit of the beneficiary. Here, the attorney was given a deed to hold and record at the appropriate time. The attorney never acquired title to the property, and the deed was in the daughter's name. Also, the attorney's duties were owed to the man; he had to hold the title and record it at the man's death unless instructed otherwise. (B) is incorrect. The statute of frauds is only one component of a valid transfer by deed. To transfer title, a deed must not only be in writing, but also be delivered. There must be words or conduct showing the grantor intended that the deed have some present operative effect-that title pass immediately and irrevocably, even though the right of possession may be postponed. Here, the man gave the deed to the attorney to record at his death "unless he gave the attorney instructions to the contrary." This language shows that the man did not relinquish control and thus there was no delivery. Without delivery, the daughter acquired no interest in the property. (D) is incorrect. The execution of a will has no effect on the testator's property because the will could be changed or revoked at any time until the testator dies. The will does not become effective until the testator dies. Thus, the execution of the man's will did not revoke the gift of the property to the daughter. Furthermore even an operative will cannot revoke a validly executed and delivered deed. A will has no effect on it because the testator no longer owns the property. Had the man delivered the deed without conditions, the property would belong to the daughter and would not pass by will to the son.
A defendant was convicted of fraud after a jury trial in state court. The conviction was affirmed on direct appeal. The defendant timely moved for postconviction relief under the Sixth Amendment on the ground that his attorney had provided ineffective assistance of counsel. The trial judge, after a hearing, found that the attorney had performed deficiently by failing to raise a proper objection that would have resulted in exclusion of important prosecution evidence. What more, if anything, must the trial court find in order to sustain the defendant's Sixth Amendment claim? Press Enter or Space to submit the answerANothing more, because the unjustifiable failure to object to important prosecution evidence is structural error.BThat the attorney was court-appointed and not privately retained.CThat there is a reasonable probability that the trial's outcome would have been different if the attorney had objected.DThat there is clear and convincing evidence that the trial's outcome would have been different if the attorney had objected.
(C) is correct. To obtain postconviction relief for ineffective assistance of counsel, the claimant must show her counsel's performance was deficient and that, but for the deficiency, the outcome of the trial would have been different. (A) is incorrect because it misstates the law. As indicated above, there must also be a showing that but for the deficiency, the result of the trial would have been different. (B) is incorrect because it does not matter if an attorney is court-appointed or privately retained; both types of attorneys are sufficient to meet the defendant's Sixth Amendment right to counsel. (D) is incorrect because it overstates the burden on the claimant for proving an ineffective assistance of counsel claim. The standard is "reasonable probability," not "clear and convincing evidence," that the outcome of the trial would have been different.
A small commercial airplane crashed in State A. The passengers and pilot, all citizens of State B, were killed in the crash. The airline that owned and operated the airplane is incorporated and has its maintenance facilities and principal place of business in State C. One day before the statute of limitations on their claims would have run, the estates of the pilot and each of the passengers filed a wrongful death action against the airline in federal court in State A. The airline was served one week later and wants to prevent the State A federal court from hearing the action. Which of the following motions is most likely to accomplish the airline's goal? Press Enter or Space to submit the answerAA motion to dismiss the action for improper venue.BA motion to dismiss the action for lack of personal jurisdiction.CA motion to dismiss the action under the doctrine of forum non conveniens.DA motion to transfer the action to a federal court in State C.
(D) is correct, and (A), (B), and (C) are incorrect for essentially similar reasoning. The best approach to answer this question is to use a process of elimination. In order to hear a case, a federal court must have subject matter jurisdiction and personal jurisdiction over the defendant, and venue must be proper. However, even if venue is proper, the court may transfer the case to another federal court, in the interests of justice, for the convenience of the parties and witnesses. Venue is proper in the judicial district in which a defendant resides, if all defendants reside in the same state, and in any judicial district where a substantial part of the acts or omissions took place or where a substantial part of the property that is the subject of the litigation is located. Here, the airplane crashed in State A, so a substantial part of the acts or omissions took place there. Thus, venue is proper, and (A) is incorrect. (B) is likewise incorrect because the airplane crashed in State A. Since the airplane crashed in State A, the airline likely committed a tort in State A, thus giving a State A court personal jurisdiction over the defendant airline under the state's long arm statute. Thus, a motion to dismiss on the basis of a lack of personal jurisdiction is unlikely to be successful. Finally, (C) is incorrect because the fact that the plane crashed in State A would likely defeat a motion to dismiss based on forum non conveniens. Federal courts typically will transfer (rather than dismiss) a case for convenience reasons under 28 U.S.C. section 1404 when the more convenient forum lies in another federal judicial district. A dismissal based on forum non conveniens usually is used when the federal court cannot transfer the action, such as when the more convenient forum is in a foreign country. Here, since all proper venues are within the United States, there is no reason the court could not transfer the case. By process of elimination, (D) is correct. The court could transfer to a federal court in State C based on convenience factors. It might be a tough sell for the airline to have the plaintiff's choice of forum overridden based on its domicile only, but it is the only possible way to have the case heard in a federal court other than in State A.
A plaintiff filed an action in federal district court and served the defendant with the summons and complaint. The defendant moved to dismiss the complaint for failure to state a claim. Instead of opposing the motion to dismiss, the plaintiff voluntarily dismissed the action and filed a new action, alleging the same claims but also addressing the pleading defects outlined in the defendant's motion to dismiss. The defendant then moved to dismiss the second action, and the plaintiff again voluntarily dismissed the second action instead of filing opposition papers. The plaintiff then filed a third action, alleging the same claims but also including additional allegations that were responsive to the defendant's second motion. The defendant has moved to dismiss the third action; the plaintiff opposes the motion. Is the court likely to grant the defendant's motion? Press Enter or Space to submit the answerANo, because the plaintiff has promptly and diligently attempted to address the pleading defects.BNo, because the plaintiff voluntarily dismissed each previous action before the defendant filed an answer or moved for summary judgment.CYes, because the plaintiff failed to seek a court order dismissing the second action.DYes, because the plaintiff's previously dismissed actions asserting the same claims operate as an adjudication on the merits.
(D) is correct. A dismissal by notice is without prejudice unless the plaintiff has previously dismissed any federal or state court action on the same claim, in which case the dismissal by notice is with prejudice. (This is known as the "two dismissal rule.") Thus, the second dismissal by the plaintiff here operates as an adjudication on the merits. (A) is an incorrect statement of the law. The rule stated above applies even if the plaintiff has acted with diligence. (B) is incorrect as well. Although the filing of an answer or a motion for summary judgment cuts off the right of the plaintiff to voluntarily dismiss without leave of court, such actions do not affect the two dismissal rule. (C) also is incorrect. Leave of court is not required if the only pending motion is a motion to dismiss.
While visiting at his son's home, a grandfather tripped on a toy left on the floor by his four-year-old grandson. The grandfather fell and was severely injured. The grandfather regularly visited his son's home and was aware that the grandson routinely left toys scattered about the house. The son had never warned the grandfather to look out for toys. The grandfather brought an action against his son to recover for his injuries. At trial, after the close of evidence, both the grandfather and the son have moved for judgment as a matter of law as to liability. The jurisdiction has abolished intra-family immunity and applies the traditional rules of landowner liability. What action should the court take? Press Enter or Space to submit the answerADeny both motions and submit the case to the jury based on negligence.BDeny both motions and submit the case to the jury based on strict liability.CGrant the grandfather's motion, because the son is liable as a matter of law for failing to warn about the risk of toys being left on the floor.DGrant the son's motion, because the son had no duty to warn that the grandson might leave toys on the floor.
(D) is correct. Under the traditional rules of landowner liability, the nature of a duty owed by an owner or occupier of land to those on the premises for dangerous conditions on the land depends on the legal status of the plaintiff with regard to the property, specifically, trespasser, licensee, or invitee. A licensee is one who enters on the land with the landowner's permission, express or implied, for her own purpose or business rather than for the landowner's benefit; social guests are considered licensees. Because the jurisdiction has abolished intra-family immunity and applies traditional rules of landowner liability, the grandfather will have the status of a licensee. The owner or occupier owes a licensee a duty to warn of or make safe a dangerous condition known to the owner or occupier that creates an unreasonable risk of harm to the licensee and that the licensee is unlikely to discover. Here, the facts state that the grandfather regularly visited his son's home and was aware that the grandson routinely left toys scattered about the house. Under these circumstances, the son had no duty to warn the grandfather about the toys and thus has not breached a duty to him. Accordingly, the son's motion should be granted. (A) is incorrect. As discussed above, the son owed no duty to the grandfather to warn him of the toys because the grandfather was aware of them. Hence, the case will not be submitted to the jury because the grandfather has not established a prima facie case of negligence. (B) is incorrect. Strict liability is not applicable here because no animals, abnormally dangerous activities, or defective products are involved. The action here would be based on negligence. (C) is incorrect. As discussed above, the son is not liable for failing to warn about the toys on the floor because the grandfather was aware that the grandson regularly left toys on the floor. Because the grandfather is a licensee, the son owed him no duty to warn.
At trial in a criminal prosecution for theft, the defendant calls a witness to testify that he formerly knew the defendant as an army supply sergeant and that the defendant had turned down many opportunities for black marketeering. Is the witness's testimony admissible? Press Enter or Space to submit the answerANo, because it is irrelevant to the present charge.BNo, because the defendant may not prove his good character by specific instances of good conduct.CYes, because a criminal defendant may prove his good character as a basis for inferring conduct.DYes, because, by accusing the defendant of being a thief, the prosecution has put his character in issue.
B) is correct. A criminal defendant may introduce evidence of his own good character for a pertinent trait to show that he did not commit the alleged crime. However, such evidence may be presented in the form of reputation and opinion testimony only; specific acts are not allowed. Thus, the defendant here may not introduce his specific acts of turning down opportunities for black marketeering to prove his good character for truthfulness. (A) is incorrect. Evidence is relevant if it has any tendency to make a fact of consequence to the determination of the action more probable or less probable than it would be without the evidence. Character evidence often meets this low bar for relevance, but is subject to various restrictions because of the danger for undue prejudice, waste of time, etc. Here, the fact that the defendant turned down opportunities for black marketeering shows his honesty and unwillingness to steal, so it is certainly relevant to the current theft charge. However, it is inadmissible because it is in the wrong form. (C) is incorrect. It is true that a defendant may introduce evidence of his own good character to show that he acted in conformity with that character during the events of the current case. However, the defendant may only introduce this type of evidence via reputation and opinion testimony. The character witness may not testify as to specific acts by the defendant. (D) is incorrect. When proof of a person's character, as a matter of substantive law, is an essential element of a claim or defense in a civil action, it is said that character is "directly in issue." When character is in issue in a case, all forms of character evidence (reputation, opinion, and specific acts) are allowed. However, character is only truly in issue in a few types of civil cases (e.g., defamation, negligent hiring and entrustment, child custody). Character is generally never directly in issue in a criminal case.
A valid treaty between the United States and a foreign country provides for the elimination of all tariff barriers between the two countries. It authorizes the president of either country to issue a proclamation nullifying any state or local laws in that country that have the effect of impeding imports from the other country. The foreign country uses the metric system of measurement, and thus all goods produced there and exported to the United States are packaged in metric sizes, such as liters and kilograms. A law of a state in the United States requires all goods sold in that state to be packaged in traditional American sizes, such as quarts or pounds. Because the state law substantially impedes imports from the foreign country, the President of the United States has issued a proclamation nullifying the state law pursuant to the treaty. Is the President's proclamation valid? Press Enter or Space to submit the answerANo, because the Constitution vests in Congress the exclusive authority to specify binding legal standards for weights and measures, and the President therefore lacks constitutional authority for the proclamation.BNo, because the principles of federalism embedded in the Constitution prohibit the President from taking action to invalidate a state law.CYes, because it is authorized by a valid treaty of the United States and is not prohibited by any provision of the Constitution and, therefore, is the supreme law of the land.DYes, because the President has inherent authority to nullify any state law that substantially impedes commerce between the United States and another country.
C) is correct. The U.S. Constitution, federal laws, and treaties are the supreme law of the land; state laws that are in conflict with the supreme law are invalid. Here, a valid treaty gives the President the power to nullify state and local laws that impede imports between the signatory nations. There is nothing in the Constitution that would prohibit such a law; indeed, the Constitution gives Congress the power to regulate commerce both among the states and with other nations. (A) is incorrect as Congress is free to delegate its authority, and here it has delegated its authority to the President. Moreover, if (A) is true and Congress has exclusive authority to specify weights and measures, the state law setting required weights and measures would be invalid anyway. (B) is simply not true. As stated above, the Constitution, federal laws, and treaties are the supreme law of the land. These can certainly give the President authority to invalidate state laws that interfere with federal objectives that are within the powers of Congress, such as objectives under the Commerce Clause. (D) is incorrect because it is overbroad. The President has no such inherent power. It would be for the courts to decide if a state law impedes commerce between the United States and another country.
A pedestrian domiciled in State A was crossing a street in State B when he was hit by a car driven by a citizen of a foreign country. Both the pedestrian and the driver suffered injuries. The pedestrian filed a negligence action against the driver in a federal district court in State B, seeking $100,000 in damages. The driver believes that the pedestrian was crossing the street illegally and is therefore responsible for the accident. The driver seeks an attorney's advice on how best to respond to the action. Assume that State B is a contributory negligence state. How should the attorney advise the driver to respond? Press Enter or Space to submit the answerAFile an answer raising the affirmative defense of contributory negligence and asserting a counterclaim for negligence, seeking damages for the driver's injuries.BFile an answer raising the affirmative defense of contributory negligence and move for judgment on the pleadings.CMove to dismiss for lack of personal jurisdiction, because the driver is not a citizen of State B.DMove to dismiss for lack of subject-matter jurisdiction, because the driver is not a U.S. citizen.
(A) is correct. Contributory negligence is an affirmative defense that must be raised in the driver's answer. Furthermore, if the pedestrian was the true cause of the action, he would be liable for the driver's damages, and thus a counterclaim would be necessary (and required, since it would be a compulsory counterclaim given that the counterclaim arose from the same transaction or occurrence as the pedestrian's complaint). (B) is incorrect because a motion for judgment on the pleadings would not be warranted. Presumably, the pedestrian was able to plead a case for negligence. It will be up to the fact-finder to determine who was truly at fault for the accident. (C) is incorrect because the driver will be subject to personal jurisdiction in State B based on a specific jurisdiction theory. Arguably, the driver was negligent by committing a tort within State B, assuming that State B has an applicable long-arm statute authorizing such jurisdiction (which all states do). (D) is incorrect because the case would fall under the court's diversity (alienage) jurisdiction. There is complete diversity between a citizen of a foreign country and a citizen of State A, and the amount in controversy requirement ($75,000) is satisfied.
A seller contracted to manufacture 1,000 toasters for a buyer for a specified price. The contract contained a provision that clearly stated: "This contract may not be assigned, and any violation of this prohibition voids the contract." After the contract was signed, the seller informed the buyer that the toasters would be manufactured by a competitor of the seller. Citing the non-assignment provision, the buyer claimed that it was no longer bound by the contract. Toasters manufactured by the competitor were of equal quality to toasters manufactured by the seller. Is the buyer bound by the contract? Press Enter or Space to submit the answerANo, because "this contract may not be assigned" means that duties may not be delegated, and the seller delegated a duty.BNo, because the seller assigned a right despite the contractual prohibition.CYes, because even though the seller breached the contract, there are no damages since the competitor's toasters are of equal quality to the seller's toasters.DYes, because the non-assignment provision is not enforceable since public policy favors free assignment and delegation.
(A) is correct. A contract clause prohibiting the assignment of the contract will be construed as barring only the delegation of the assignor's duties. Here, the seller delegated its duty to manufacture the toasters to a competitor. This voided the contract, and the buyer is no longer bound. (B) is incorrect. The seller delegated its duty to manufacture the toasters to a competitor. It did not assign its right to payment, which would have been permissible. (C) is incorrect because, as stated above, the manufacturer's delegation of its duties rendered the contract void. (D) is also incorrect. Nonassignment provisions will be enforced, but absent circumstances suggesting otherwise, a clause prohibiting the assignment of the contract will be construed as barring only the delegation of the assignor's duties.
A woman sued her former employer in state court, asserting age and sex discrimination claims under both state and federal law. The woman's attorney had recently been embarrassed in court by the judge to whom the case was assigned. Wishing to avoid difficulties with the judge, the woman's attorney promptly removed the case to federal court on the basis of federal-question jurisdiction. The employer's attorney has timely moved to remand. How is the federal court likely to proceed? Press Enter or Space to submit the answerARemand the entire case.BRemand the state claims but keep the federal claims.CRetain the case to avoid the risk of bias and impropriety in having it proceed before a judge who has shown clear hostility toward the woman's attorney.DRetain the case, because it was timely removed and the woman alleges federal claims.
(A) is correct. Although federal subject matter jurisdiction exists based on the presentation of a federal question, the woman is not allowed to remove because she is the plaintiff, and plaintiffs are not allowed to remove a case. Thus, the entire case must be remanded due to this defect. (B) is incorrect. Because the plaintiff is not allowed to remove, the entire case must be remanded. Note that there is a rule that when a case filed in state court contains a claim that would arise under federal law, and it is joined with state law claims that do not invoke diversity or supplemental jurisdiction, the entire case can be removed to federal court. The federal court, however, must then sever and remand the purely state law claims to state court. (C) is incorrect. Local prejudice does not provide a basis for removal. (D) is incorrect because it ignores the fact that plaintiffs have no right of removal.
The builder of a new house sold the house to a buyer, conveying it to the buyer by warranty deed. One year later, the buyer sold the house to a woman for $50,000 more than the buyer had paid, conveying it to the woman by a quitclaim deed. Four months after moving in, the woman discovered a number of latent defects in the foundation that caused the house's basement to take on water during heavy rains. The woman contacted the builder about the problem. Toward the end of their heated conversation, the builder yelled at her, "So get it fixed!" After the woman had the foundation repaired by a cement contractor at a cost of $25,000, she successfully sued the builder to recover that amount from him. What is the most likely reason that the woman was successful? Press Enter or Space to submit the answerAThe court found an implied warranty of habitability from the builder to the buyer that was enforceable by a subsequent buyer.BThe court found that by paying the buyer $50,000 more for the house than the buyer had paid the builder, the woman did not get the benefit of her bargain.CThe court found that by yelling at the woman to "get it fixed," the builder had committed himself to paying for the repair.DThe court found that the defects in the foundation were a breach of the covenant of further assurances in the warranty deed from the builder to the buyer.
(A) is correct. Although the common law held that contracts of sale and deeds of real property carry no implied warranties of quality or fitness for the purpose intended, most courts now find an implied warranty of fitness or quality extends to the sale of any new house by the builder. The warranty implied is that the new house is designed and constructed in a reasonably workmanlike manner and suitable for human habitation. Some courts go further and extend the warranty to a subsequent purchaser. Here, there was a defect in the design or construction of the house by the builder, and the most likely reason the woman would succeed here is that the court recognizes that the builder's warranty of fitness or quality applies to her, a subsequent purchaser. (B) is incorrect. Whether the woman got the benefit of her bargain in her contract with the buyer would have no impact on whether she could recover from the builder. The woman's contract was with the buyer, not the builder. The only way the buyer, who was not in privity with the builder, could recover from him would be if the implied warranty given to the buyer is extended to the woman. (C) is incorrect. The builder's statement to "get it fixed" does not clearly indicate that he would pay for the repairs. It could just as easily be seen as a statement that it was her problem and her responsibility. (D) is incorrect. The covenant for further assurances in a warranty deed is a covenant to perform whatever acts are reasonably necessary to perfect the title conveyed if it turns out to be imperfect. The defect in the foundation did not make title imperfect; title to the property was good. The covenant for further assurances does not apply.
Under the authority of a federal voting rights statute, some states drew congressional districts in a manner calculated to increase the likelihood that members of historically disadvantaged minority racial groups would be elected. The U.S. Supreme Court declared these districts to be unconstitutional, as improper racial gerrymanders. In response to this ruling, Congress passed a new statute that explicitly denies the Supreme Court appellate jurisdiction over all future cases challenging the constitutionality of action taken under the authority of the federal voting rights statute. Which of the following is the most persuasive argument for the constitutionality of the new statute restricting the Supreme Court's appellate jurisdiction? A Article III of the Constitution explicitly states that the Supreme Court's appellate jurisdiction is subject to such exceptions and regulations as Congress shall make.BThe constitutional principle of separation of powers authorizes Congress to pass statutes calculated to reduce the effects of Supreme Court decisions that interfere with the exercise of powers that have been delegated to the legislative branch.CThe establishment and apportionment of congressional districts directly affect interstate commerce, and the Constitution authorizes Congress to use its plenary authority over such commerce for any purpose it believes will promote the general welfare.DThe Fifteenth Amendment authorizes Congress to enforce the amendment's voting rights provisions by appropriate legislation, and Congress could reasonably determine that this restriction on the Supreme Court's appellate jurisdiction is an appropriate means to that end.
(A) is correct. Article III explicitly gives Congress the power to make exceptions to the Supreme Court's appellate jurisdiction. (B) is incorrect. The separation of powers doctrine is described as a series of checks and balances among the three branches of government - just the opposite of what this choice suggests. (C) is incorrect. Although Congress has broad powers under the Commerce Clause, they are not without limit. The Commerce Clause gives Congress power to adopt laws concerning channels of interstate commerce, such as roads and airways; instrumentalities of interstate commerce, such as trucks and trains; and economic or commercial activities - even ones that take place solely within one state - that in aggregate have a substantial effect on interstate commerce. Congressional voting districts do not fall within any of these categories. Moreover, the General Welfare Clause is part of Congress's spending power (it may spend for the general welfare) and is not part of Congress's commerce power. (D) is incorrect because it is not as strong an answer as (A). Article III explicitly allows Congress to make exceptions to the Supreme Court's appellate jurisdiction without qualification. The enabling clause of the Fifteenth Amendment allows Congress to adopt legislation protecting the right to vote from discrimination. A law taking away from the Supreme Court jurisdiction to hear cases under a voting rights statute would seem to be the opposite of what is allowed by the Fifteenth Amendment's enabling clause.
An architect agreed with a developer to design a large residential development. Because the architect had a history of substance abuse problems, the parties agreed that the developer's duty to accept and pay for the plans was conditioned on the architect's abstaining from drinking alcohol during the six months it would take to do the work. After two months, the architect began having several alcoholic drinks each day. The developer became aware of the architect's drinking before the architect showed the developer the preliminary plans. When the developer saw the preliminary plans, the developer told the architect that the concept was impressive and that he looked forward to seeing the final plans. The architect continued to have several alcoholic drinks each day and completed the plans within the specified contract period. However, the developer declined to review, accept, or pay for the final plans and stated that it was because of the architect's continued use of alcohol. The architect has sued the developer for breach of contract. Which of the following arguments best supports the architect's claim? Press Enter or Space to submit the answerAThe alcohol use was not a material breach of contract by the architect, since it did not affect the quality of the work.BThe developer waived the condition of no alcohol use.CThe no-alcohol term was functionally a penalty clause and therefore was unenforceable.DThe no-alcohol term would be interpreted as a promise and not a condition.
(B) is correct. A contract may provide that a party does not have a duty to perform unless some condition is fulfilled. Here the contract provided that the developer's duty to accept and pay for the plans was conditioned on the architect abstaining from drinking alcohol during the creation of the plans. The architect failed to do this, so the condition was not fulfilled. However, one having the benefit of a condition can waive that condition by indicating by words or conduct that he will not insist on that condition's being met. If the party waives the condition, that party must perform despite the condition's failure. Consideration is not required for a valid waiver of condition. The architect's best argument is that the developer waived the condition when the developer became aware that the architect was drinking, but still said he was looking forward to the final plans. If the developer waived the condition, he retained his duty to perform, and the architect would be successful in suing the developer for breach. (A) is not the architect's best argument. If the architect argues that his alcohol use was a nonmaterial breach of contract, rather than a condition that was waived, he would be entitled to be paid under the contract, but the developer would be entitled to offset damages from the breach. (C) is also not the best argument for the architect. Punitive damages are not enforceable under contract law. However, the no-alcohol term does not appear to be a penalty. The courts would consider the reasonableness of the term, and under the circumstances (the architect had a history of substance abuse), it was reasonable for the developer to insist on the architect's sobriety. (D) is incorrect. It would not be beneficial to the architect to argue the no-alcohol term was a promise rather than a condition, since a condition, unlike a promise, can be waived without consideration and breach of promise, like the nonmaterial breach discussed above, would make the architect liable for damages. Moreover, whether any given contractual provision is to be interpreted as a promise or condition is determined by the "intent of the parties." Here, the words of the agreement ("conditioned on") indicate that an express condition was intended.
A man has sued a police officer, alleging that the officer violated the man's civil rights by using excessive force while arresting him. At trial, the officer admits having hit the man in the head with the butt of his gun, but contends that the force was necessary, because the man was resisting arrest. In support of his contention, the officer seeks to introduce evidence that the man had resisted arrest on three prior occasions during the last 10 years. Is this testimony regarding the man's conduct during the three prior arrests admissible? Press Enter or Space to submit the answerANo, because evidence of the prior incidents constitutes impermissible character evidence.BNo, because the officer has not shown that the man was convicted in connection with the prior incidents.CYes, because the incidents in question are relevant evidence of the man's propensity for violence.DYes, because the incidents in question are sufficient to constitute a habit.
(A) is correct. Evidence of a person's other acts is generally inadmissible to show how a person probably acted on a particular occasion. Although such evidence may be admitted if relevant for an independent purpose, here the evidence only tends to show that the man has a propensity to resist arrest. The officer is seeking to show that because the man resisted arrest on the prior occasions, he is more likely to have resisted arrest during the events of this case. Consequently, it is inadmissible character evidence. (B) is incorrect. The evidence would be inadmissible to show the man's propensity to resist arrest even if the man had been convicted of the prior acts. And even though evidence of certain prior convictions may be introduced to impeach a witness, there is no indication that the man has testified at this trial. (C) is incorrect. A person's other acts are generally not admissible to show the person's propensity to act a certain way. Thus, the evidence cannot be admitted for this purpose. (D) is incorrect. Habit evidence is evidence of a person's regular response to a specific set of circumstances. It is admissible to prove that the person acted in accordance with the habit on a particular occasion. However, resisting arrest three times over a span of 10 years is not frequent enough to constitute habit.
A defendant has been charged with making a false statement to a federally insured financial institution to secure a loan. At trial, the prosecutor calls the defendant's wife as a willing witness to testify that the defendant told her in confidence that he had misrepresented his assets on the loan application. The defendant objects to his wife's testimony. Should the testimony be admitted? Press Enter or Space to submit the answerANo, because even though the wife is a willing witness, the defendant has the right to exclude confidential marital communications in federal court.BNo, but only if the law of the state where the defendant and his wife reside recognizes a privilege for confidential marital communications.CYes, because in federal court the right not to testify belongs to the testifying spouse, and she is a willing witness.DYes, because while the adverse testimonial privilege is recognized in federal court, the marital communications privilege is not.
(A) is correct. Federal courts recognize two privileges arising out of the marital relationship: (i) the spousal testimonial privilege (also known as the doctrine of spousal immunity), which prevents a spouse from being compelled to testify against her spouse in a criminal case; and (ii) the privilege for confidential marital communications, which applies in both criminal and civil cases, survives termination of the marriage, and may be claimed by either spouse. Here, the prosecutor wants to introduce testimony by the defendant's wife as to what he told her in confidence; thus, the defendant may invoke the confidential marital communications privilege to prevent the wife from testifying about their conversation. (B) is incorrect. In diversity cases, federal courts apply state privilege law. However, this is a federal question case arising out of the defendant's false statement to a federally insured financial institution. Federal courts recognize the privilege for confidential marital communications, so the defendant may invoke this privilege irrespective of state law. (C) is incorrect. The spousal testimonial privilege belongs to the witness-spouse only. However, the separate privilege for confidential marital communications belongs to both spouses. Because the wife's testimony concerns their private conversation, the defendant may invoke the marital communications privilege and prevent her from testifying about the conversation. (D) is incorrect. Federal courts recognize both the spousal testimonial privilege and the privilege for confidential marital communications.
An elderly woman underwent major surgery and spent two weeks in the hospital. The woman continued to take powerful pain medication for several weeks after she returned home. During her recovery, she offered to sell her car for $450 to her neighbor, who owned a house-cleaning service. The neighbor said, "That's great! I need a car to transport all the people who work for me to their job sites." In fact, the woman's car was worth $3,000, and the neighbor knew this. He was also aware that the woman had undergone surgery and noted that she seemed "out of it" because of the medication she was taking. Several days later, the woman's son found out about the deal and contacted the neighbor, telling him that the woman would sell him the car, but for $3,450. The next day, when the neighbor tendered $450 and demanded that the woman give him the car, she refused. If the neighbor sues the woman for breach of contract, will he be likely to prevail? Press Enter or Space to submit the answerANo, because the contract was voidable due to the woman's apparent incapacity.BNo, because the woman put nothing in writing.CYes, because the neighbor's reliance on the otherwise voidable contract made it enforceable.DYes, because the woman's offer and the neighbor's acceptance created an enforceable contract.
(A) is correct. For a contract to be enforceable, both parties must have had capacity to enter into a contract. One who is so intoxicated by drugs or alcohol that she does not understand the nature and significance of her promise may be held to have made only a voidable promise if the other party had reason to know of the intoxication. Here, the woman was under the influence of powerful pain medication at the time she made the offer to sell her car to the neighbor. The neighbor was aware of her incapacity, noting that she seemed "out of it." The woman since refused to affirm her voidable promise, so it is likely that the neighbor will be unable to prevail in a breach of contract action against her. (B) is incorrect. The neighbor is unlikely to prevail due to the woman's incapacity, not because the contract was not in writing. This contract did not need to be in writing to be enforceable. A contract for the sale of goods for a price of $500 or more is within the Statute of Frauds and generally must be evidenced by a signed writing to be enforceable. The contract here was for only $450, so a writing was not required to make the contract enforceable. It is the price in the contract and not any other perceived value of the goods that matters. (C) is incorrect. The neighbor was aware of the woman's incapacity at the time he made the contract, and there is no evidence of reliance here. The neighbor's need for a car did not change based on the woman's promise. (D) is incorrect. As stated above, due to the woman's incapacity, and the neighbor's awareness of that incapacity, the woman's promise was voidable. Since the woman now refuses to go through with the sale, there is no enforceable contract.
An individual investor purchased stock through a company's stock offering. When the price of the stock plummeted, the investor sued the company in a state court in State A, claiming that the company's offering materials had fraudulently induced him to purchase the stock and seeking $25,000 in damages. A university that had purchased the company's stock through the same offering sued the company in federal court in State B, claiming that the offering materials violated federal securities laws and seeking $1 million in damages. The individual investor's suit proceeded to trial. The state court ruled that the company's offering materials contained false information and awarded the investor a $25,000 judgment. The university immediately moved for partial summary judgment in its federal action against the company, arguing that the state court judgment bound the federal court on the issue of whether the company's offering materials contained false information. Neither State A nor State B permits nonmutual issue preclusion. Should the court grant the university's motion? Press Enter or Space to submit the answerANo, because State A does not permit nonmutual issue preclusion.BNo, because the federal court sits in a state that does not permit nonmutual issue preclusion.CYes, because federal law permits nonmutual issue preclusion.DYes, because the issue of whether the materials contained false information was actually litigated and necessarily decided.
(A) is correct. Generally speaking, when dealing with the preclusive effect of a judgment, the recognizing court should not give a judgment any greater effect than the rendering state would. (When "case one" has been decided in state court, the court in case two generally will apply the claim or issue preclusion of the jurisdiction that decided case one.) Here, the rendering state, State A, does not recognize nonmutual collateral estoppel, so a federal court sitting in State B should not give the judgment issue preclusive effect. (B) is incorrect. As stated above, the law of the rendering state generally determines the preclusive effect of the judgment. (C) is incorrect for a similar reason. The fact that federal law may permit nonmutual issue preclusion is irrelevant. The law of the rendering state generally controls the preclusive effect of the judgment. (D) is incorrect. The listed factors are used to determine whether a court may apply issue preclusion, but they do not address whether nonmutual issue preclusion is applicable.
A football team entered into a 10-year lease with a city for use of the city's athletic stadium. Five years into the lease, the team threatened to leave the stadium and move to another city. The city sued the team in federal court, seeking a permanent injunction to prevent the team from breaching its lease and leaving. In its answer, the team included a counterclaim seeking $10 million in damages for losses caused by the city's alleged failure to properly maintain the stadium, as the lease required. The team demanded a jury trial on the counterclaim. The city moved to try its claim for a permanent injunction before the trial on the team's counterclaim. The team objected and moved that the jury trial of its counterclaim be held before the trial of the city's injunction claim. How should the court rule on the parties' motions? Press Enter or Space to submit the answerAThe court should first hold a jury trial of the team's counterclaim, and then a nonjury trial of the issues remaining in the city's claim.BThe court should first hold a nonjury trial of the city's claim without giving binding effect to its findings or conclusions in the later jury trial of the team's counterclaim.CThe court should first hold a nonjury trial of the city's claim, and then a jury trial of the issues remaining in the team's counterclaim.DThe court should schedule a jury trial of both the city's claim and the team's counterclaim.
(A) is correct. If legal and equitable claims are joined in one action involving common fact issues, the legal claim should be tried first to the jury and then the equitable claim to the court, but the jury's finding on fact issues will bind the court in the equitable claim. (B) is incorrect because only one trial is held. As stated, the jury's findings on fact issues will bind the court in the equitable claim. (C) is incorrect because a party cannot be denied a jury trial on "legal" issues even though there may be mixed "legal" and "equitable" claims in the case. (D) is incorrect because a party is not entitled to a jury on equitable claims.
A construction contractor brought a breach of contract claim in federal court against a homeowner who had hired the contractor to build an apartment over an existing garage. The action turned on the scope of the work covered by the contract. The contractor and the homeowner were the only witnesses at the bench trial, and they strongly disagreed about the scope of the work. At the end of the trial, the judge stated findings of fact on the record but never issued a written opinion. Neither party objected to the findings. The judge found in favor of the homeowner, and the contractor appealed. Is the appellate court likely to overturn the findings? Press Enter or Space to submit the answerANo, because the appellate court must give due regard to the trial judge's opportunity to determine witness credibility.BNo, because the contractor failed to object to the findings when the judge stated them in open court.CYes, because a judge must set forth findings of fact in a written opinion or memorandum of decision.DYes, because there were disputed issues of fact at trial.
(A) is correct. In a bench trial, the judge acts as the fact-finder, and the appellate court will give deference to the judge's findings of fact. There are no facts in the question to raise any question about the judge's findings, and he must have found the homeowner to be the more credible witness. (B) is incorrect. An objection to the judge's findings of fact is not required. (C) is incorrect, as it is an incorrect statement of the law. Under Federal Rule of Civil Procedure 52, the judge's findings of fact (and conclusions of law) may be stated on the record after the close of the evidence or may appear in an opinion or a memorandum of decision filed by the court. (D) is incorrect. One would expect at least some facts to be disputed at trial; the fact that facts are disputed does not override the policy that the person who hears the evidence is in the best position to assess the evidence's credibility.
A plaintiff sued the insurer of her home after the insurer denied coverage for water damage to the home allegedly caused by a frozen plastic pipe that burst. At trial, the insurer called as an expert witness an engineer, who testified that the pipe had burst because of age rather than freezing. On cross-examination, the engineer admitted that, five years earlier, he had been convicted of tax fraud, even though he had asserted that it was his accountant's error. In response, the insurer calls a witness, who is well acquainted with the engineer and his reputation, to testify that (1) in the witness's opinion, the engineer is a truthful person, and (2) the engineer's neighbors all describe him as a truthful person. How much, if any, of the witness's testimony is admissible? Press Enter or Space to submit the answerAAll of the testimony is admissible to support the engineer's credibility.BOnly the portion concerning the engineer's reputation is admissible, because where both opinion and reputation evidence are available, only the latter is admissible under a rule of preference.COnly the portion concerning the witness's opinion of the engineer's character, because the witness's reporting of the neighbors' comments is hearsay.DNone of the testimony is admissible, because it is collateral, having no bearing on the engineer's qualifications as an expert.
(A) is correct. Once a witness is impeached with evidence of his bad character for truthfulness, he may be rehabilitated with reputation and opinion evidence of his good character for truthfulness. Here the plaintiff has attacked the engineer's character for truthfulness by presenting evidence of his prior tax fraud conviction, so the insurer may now present evidence to rehabilitate the engineer. The witness's opinion and reputation testimony as to the engineer's good character for truthfulness is proper. (B) is incorrect. Reputation and opinion evidence are both admissible to rehabilitate a witness and there is no rule of preference favoring one over the other. (C) is incorrect. Although reputation evidence of character is hearsay in a sense-it is a compilation of out-of-court statements by those familiar with the person's reputation-there is a hearsay exception for reputation evidence of a person's character. (D) is incorrect. Whether the engineer is a truthful person has bearing on his credibility, and the credibility of a witness is never a collateral matter.
Police officers had probable cause to believe that drug dealing was routinely taking place in a particular room at a local motel. The motel manager authorized the officers to enter the room and provided them with a passkey. Without obtaining a warrant, the officers knocked on the room's door, announced their presence, and told the occupants that they would like to speak with them. The officers then heard yelling and repeated flushing of the toilet. They then used the passkey and entered the room, where they saw the occupants dumping drugs into the toilet. The occupants of the room were charged with drug dealing and have moved to suppress the drugs. Should the court grant the motion to suppress? Press Enter or Space to submit the answerANo, because exigent circumstances justified the officers' entry.BNo, because the motel manager consented to the officers' entry.CYes, because exigent circumstances cannot excuse the lack of a warrant.DYes, because the officers cannot benefit from exigent circumstances that they created.
(A) is correct. The Fourth Amendment prohibits unreasonable searches and seizures. As a general rule, warrantless searches and seizures in constitutionally protected areas are per se unreasonable absent an exception to the warrant requirement. Exigent circumstances are one such exception. If exigent circumstances exist, such as imminent destruction of evidence, the police may enter constitutionally protected premises without a warrant and seize the evidence to prevent its destruction. And this is true even if the exigency was created by the police-so long as it was not created in actual or threatened violation of the Fourth Amendment. Here, the exigency (the sounds of yelling and toilet flushing after police officers knocked on the door and asked for entry, suggesting that drugs were being destroyed) did not arise through a violation or threatened violation of the Fourth Amendment. Police officers may knock on a door to ask questions just like any member of society can-no warrant is required. Therefore, the officers could lawfully enter the motel room. For those reasons, (C) and (D) are incorrect. (B) is incorrect because the motel manager did not have actual or apparent authority to consent to the search of the occupied room. Overnight guests of a motel have a reasonable expectation of privacy in their room, which functions like a temporary residence. The officers could not reasonably assume the manager could authorize their entrance while the room was being occupied. In this case, the legal exception to the warrantless entry is the exigent circumstances, not consent.
A man became ill while at work and decided to go home early. When he entered his bedroom, the man saw his wife engaged in sexual intercourse with a neighbor. The man grabbed a gun from a dresser drawer and shot and killed the neighbor. He was later charged and prosecuted. In a jurisdiction that follows the common law for homicide offenses, which crimes should the court instruct the jury on? Press Enter or Space to submit the answerAMurder and involuntary manslaughter.BMurder and voluntary manslaughter.CMurder, voluntary manslaughter, and involuntary manslaughter.DVoluntary manslaughter and involuntary manslaughter.
(B) is correct. The court should instruct the jury on murder and voluntary manslaughter. Murder is the unlawful killing of another human being with malice aforethought. Malice aforethought exists if the defendant has one of the following states of mind: (i) the intent to kill, (ii) the intent to inflict great bodily injury, (iii) reckless indifference to an unjustifiably high risk to human life, or (iv) the intent to commit a felony. Here, the man grabbed a gun and shot and killed the neighbor. A jury could find that he had the intent to kill, or at least the intent to inflict great bodily injury. The court should therefore instruct the jury on murder. Voluntary manslaughter is a killing that would otherwise be murder but is distinguishable from murder by the existence of adequate provocation. Provocation will reduce a killing to voluntary manslaughter if it meets four tests: (i) the provocation must have been one that would arouse sudden and intense passion in the mind of an ordinary person such as to cause him to lose his self-control; (ii) the defendant must have in fact been provoked; (iii) there must not have been a sufficient time between the provocation and the killing for the passions of a reasonable person to cool; and (iv) the defendant in fact did not cool off between the provocation and the killing. Adequate provocation is frequently recognized in the case of one spouse discovering the other in bed with another person. Here, the man's actions meet the requirements for voluntary manslaughter. He caught his wife in bed with the neighbor, which would provoke an ordinary person and provoked the man himself. It appears that there were only a few seconds between the provocation and killing, which would be insufficient time for a reasonable person to cool, and nothing indicates that the man cooled off. Because the man's actions fit the requirements for voluntary manslaughter, the jury should be instructed on that crime. If the jury were to find that this killing did not qualify as voluntary manslaughter, the man would be guilty of common law murder. Thus the jury should be instructed on both crimes. (A) is incorrect. The killing does not meet the requirements of involuntary manslaughter. There are two types of involuntary manslaughter: (i) a death caused by criminal negligence and (ii) a killing caused by an unlawful act, either a misdemeanor or a felony not included in felony murder. Here, there is no indication that the man was negligent; his actions appear to be intentional. Also, the man did not cause the neighbor's death while committing an unlawful act; he merely came home early from work. Because the killing does not fit into the definitions of involuntary manslaughter, the court should not instruct the jury on it. (C) is incorrect. As discussed above, the killing does not meet the requirements for involuntary manslaughter, and the jury should not be instructed on it. (D) is incorrect. As explained above, the court should instruct the jury on murder but not on involuntary manslaughter.
Without a warrant, police officers searched the garbage cans in the alley behind a man's house and discovered chemicals used to make methamphetamine, as well as cooking utensils and containers with the man's fingerprints on them. The alley was a public thoroughfare maintained by the city, and the garbage was picked up once a week by a private sanitation company. The items were found inside the garbage cans in plastic bags that had been tied closed and further secured with tape. The man was charged in federal court with the manufacture of methamphetamine. Did the search of the garbage cans violate the Fourth Amendment? Press Enter or Space to submit the answerANo, because the man had no reasonable expectation of privacy in garbage left in the alley.BNo, because the probative value of the evidence outweighs the man's modest privacy claims in his garbage.CYes, because the alley was within the curtilage of the man's home and entry without a warrant was unconstitutional.DYes, because there is a reasonable expectation of privacy in one's secured garbage containers.
(A) is correct. The Fourth Amendment prohibits unreasonable searches and seizures. It is implicated when the government searches or seizes something in which the person subject to the search has a reasonable expectation of privacy. One does not have a reasonable expectation of privacy in things held out to the public. The Supreme Court has held that one has a reasonable expectation of privacy in one's home and the home's curtilage (that is, the land and outbuildings immediately adjacent to the home); however, one does not have a reasonable expectation of privacy in garbage when it has been put out for collection outside the home's curtilage because there is no reasonable expectation of privacy in abandoned property. For that reason, (D) is incorrect. (B) is incorrect because it does not state a legal standard for the admissibility of evidence in criminal proceedings. The question of admissibility is dependent on the legality of the search and seizure, not the probative weight of the evidence. (C) is incorrect because, as a public thoroughfare, the alley does not constitute the curtilage of a home.
A federal statute authorizes a federal agency to issue rules requiring that state legislatures adopt laws of limited duration to reduce water pollution from gasoline-powered boat motors. The purpose of these rules is to assist the agency in attaining the clean water standards required by the statute. After the agency issued such rules, several states filed an action challenging the rules on the sole ground that they are unconstitutional. Should the court uphold the constitutionality of the agency's rules? Press Enter or Space to submit the answerANo, because the federal government may not compel a state legislature to enact into state law a federally mandated regulatory program.BNo, because the Tenth Amendment grants states immunity from all direct federal regulation.CYes, because the rules serve an important purpose, and the requirements they impose on the states are only temporary and do not excessively interfere with the functioning of the state governments.DYes, because the Supremacy Clause of Article VI requires states to enforce federal law.
(A) is correct. The Supreme Court has held that the Tenth Amendment prohibits the federal government from adopting a statute that compels the states to enact or enforce a regulatory program. Under our system of federalism, the federal government cannot order state governments to adopt particular laws. (B) is incorrect. It is overbroad. The Tenth Amendment provides that powers not delegated to the United States nor prohibited to the states are reserved to the states. The Supreme Court has held that the amendment generally prohibits the federal government from regulating states alone. However, states can be regulated through laws that apply generally. For example, under Congress's commerce power, the federal government can require state governments to pay employees the federal minimum wage, like any other employer. Moreover, it has been held that under Congress's powers under the Fourteenth and Fifteenth Amendments, the federal government may restrict state activities that the federal government determines would violate civil liberties. (C) is incorrect. This choice appears to be applying some sort of intermediate scrutiny test, which seems reasonable. However, no such test applies. (D) is incorrect. It is true that the Supremacy Clause requires states to enforce federal law - but only legitimate federal law. As indicated above, the federal government does not have the power to order state governments to enact a regulatory scheme. Neither does the federal government have power to commandeer state officials.
A credit card company obtained and properly filed a judgment against a man after he failed to pay a $10,000 debt. A statute in the jurisdiction provides as follows: "Any judgment properly filed shall, for 10 years from filing, be a lien on the real property then owned or subsequently acquired by any person against whom the judgment is rendered." Two years later, the man purchased land for $200,000. He made a down payment of $20,000 and borrowed the remaining $180,000 from a bank. The bank loan was secured by a mortgage on the land. Immediately after the closing, the deed to the man was recorded first, and the bank's mortgage was recorded second. Five months later, the man defaulted on the mortgage loan and the bank initiated judicial foreclosure proceedings. After receiving notice of the proceedings, the credit card company filed a motion to have its judgment lien declared to be the first lien on the land. Is the credit card company's motion likely to be granted? Press Enter or Space to submit the answerANo, because the bank's mortgage secured a loan used to purchase the land.BNo, because the man's down payment exceeded the amount of his debt to the credit card company.CYes, because the bank had constructive notice of the judgment lien.DYes, because the bank is a third-party lender and not the seller of the land.
(A) is correct. The bank's mortgage is a purchase-money mortgage, meaning that the funds the bank advanced were used to purchase the land. A purchase-money mortgage executed at the same time as the purchase of the real property encumbered takes precedence over any other claim or lien, including a previously filed judgment lien. Therefore, the bank's purchase-money mortgage takes precedence over the credit card company's judgment lien. (B) is incorrect. The relative amounts of the down payment and the credit card debt are irrelevant. Regardless of the amounts, a purchase-money mortgage executed at the same time as the purchase of the real property encumbered takes precedence over any other claim or lien, including a previously filed judgment lien. (C) is incorrect. It is true that the judgment lien was properly filed and thus provided the bank with constructive notice of the lien. However, as discussed above, a purchase-money mortgage executed at the same time as the purchase of the real property encumbered takes precedence over any other claim, including a previously filed judgment lien. (D) is incorrect. The bank's mortgage is a purchase-money mortgage, which may be granted by a seller, by a third party, or both. As discussed above, the bank's purchase-money mortgage takes precedence over the credit card company's judgment lien.
A defendant, a nurse at a nursing home, is charged with murdering a resident at the home by adding an allegedly lethal substance to the resident's food. At trial, to prove that the substance added to the resident's food could result in death, the prosecutor, without first calling any witnesses, offers to read into evidence several pages from a standard medical treatise that support the prosecution's claim that the substance the defendant added to the food is lethal. Is the evidence offered admissible? Press Enter or Space to submit the answerANo, because the treatise excerpts were not offered during the examination of a qualified expert.BNo, because the treatise itself must be introduced as an exhibit.CYes, although hearsay, under the learned treatise exception to the hearsay rule.DYes, because the lethal nature of the substance is relevant to the defendant's state of mind and intent.
(A) is correct. The content of the treatise is hearsay because it is an out-of-court statement being offered for the truth of its contents. There is a hearsay exception that allows portions of learned treatises to be read into evidence, but this exception applies only when the treatise is being used on direct or cross-examination of an expert witness. Here the prosecutor is trying to read portions of the treatise before calling any expert witness to the stand, so the treatise is inadmissible hearsay. (B) is incorrect. Even when the hearsay exception for learned treatises does apply, the treatise is read into evidence only-it is not received by the jury as an exhibit. (C) is incorrect. As stated above, the learned treatise hearsay exception does not apply here because the prosecutor is not using the treatise in the context of expert testimony. (D) is incorrect. Although the lethal nature of the substance is certainly relevant to show the defendant's state of mind and intent, the treatise is inadmissible hearsay.
A homeowner contracted in writing with a kitchen contractor to renovate her kitchen for $25,000, "subject to the homeowner's complete personal satisfaction." The contractor replaced the cabinets, flooring, and countertops and then sought payment from the homeowner. The homeowner paid the contractor only $20,000, truthfully saying that she did not like the finish on the cabinets and was therefore not satisfied. If the contractor sues the homeowner for the balance of the contract price, will the contractor be likely to prevail? Press Enter or Space to submit the answerCorrectANo, because a condition to the homeowner's obligation to pay was not satisfied.BNo, because the contractor breached his duty of good faith and fair dealing by supplying unsatisfactory materials.CYes, because the homeowner breached the covenant of good faith and fair dealing by rejecting the cabinets without justification.DYes, because the homeowner was the first party to breach the contract.
(A) is correct. The contract contained an express condition that the homeowner's performance would become due only if she was satisfied with the results of the renovation. This was a condition precedent. A condition precedent must occur before an absolute duty of immediate performance arises in the other party. A contract of satisfaction that involves matters of personal taste, such as this one, is fulfilled only if the promisor is personally satisfied. If the promisor claims that she is not satisfied, her lack of satisfaction must be honest and in good faith. Since here the homeowner honestly was not satisfied, the condition precedent did not occur, and the homeowner has no duty to perform under the contract. (B) reaches the right conclusion but states the wrong reason. The issue is the homeowner's satisfaction. The contractor did not breach any duties - the contractor performed - but the condition precedent to the homeowner's performance coming due did not occur. (C) is incorrect. The homeowner was truthful when she said that she did not like the finish on the cabinets. As stated above, so long as her lack of satisfaction was honest and in good faith, the express condition requiring her personal satisfaction did not occur. (D) is wrong. Neither party breached the contract. This question is dealing with a failure of a condition precedent, not breach.
A homeowner and a contractor entered into a contract under which the homeowner agreed to pay the contractor $50,000 for remodeling the homeowner's basement according to a set of plans. After the work was completed, the homeowner honestly believed that there were defects in the contractor's work as well as departures from the plans. In fact, the contractor had fully performed. The homeowner offered to pay the contractor $35,000 in full settlement in exchange for the contractor's promise to surrender his entire claim. The contractor accepted the homeowner's offer, and the homeowner paid the contractor $35,000. The reasonable value of the work was $35,000. Is the contractor likely to succeed in an action challenging the validity of the settlement agreement? Press Enter or Space to submit the answerANo, because the homeowner honestly disputed the amount he owed the contractor.BNo, because the reasonable value of the work was only $35,000.CYes, because the contractor reasonably relied on the homeowner's contractual promise to pay the full $50,000, and that promise should be enforced to avoid injustice.DYes, because the homeowner's payment of $35,000 cannot furnish consideration for the contractor's relinquishment of a claim for $50,000.
(A) is correct. The homeowner's promise to surrender his claim against the contractor was sufficient consideration for a modification of the contract price. A common law contract (such as the one here - for services) can be modified if the modification is supported by new consideration. The promise to refrain from suing on a claim may constitute consideration. Even if the claim is invalid, in law or in fact, if the claimant reasonably and in good faith believes his claim to be valid, forbearance of the legal right to have the claim adjudicated constitutes a detriment and consideration. Here, the homeowner honestly believed that there were defects in the contractor's work and departures from the plans. Even though the homeowner's belief is incorrect, and thus he is unlikely to win a suit against the contractor, this belief is in good faith, so a surrender of his right to sue on the claim constitutes valid consideration sufficient to modify the contract price. (B) is incorrect because it gives the wrong reason for why the contractor is unlikely to succeed. The reasonable value of the work is irrelevant here. Courts of law normally will not inquire into the adequacy of consideration. The parties to a contract are free to set their own price. The court will enforce the contract as modified because that is the price agreed to by the parties, without considering the merits of that price. (C) is incorrect. While it is true that the contractor relied on the initial promise of $50,000 by remodeling the basement, parties are free to modify their agreements, and the parties here did so, as explained above. There is no injustice in enforcing the contract as modified. (D) is incorrect because as stated above, the homeowner's surrender of his claim is sufficient consideration for modification of the contract price.
A plaintiff, a management trainee, brought a sex discrimination lawsuit against her employer for wrongful termination of her employment. At trial, the plaintiff is prepared to testify that a janitor at the company told her that he had heard her supervisor say to other male co-workers about her, "Make it hard for her. Maybe she'll go home where she belongs." Is the plaintiff's proposed testimony admissible? Press Enter or Space to submit the answerANo, because the janitor's statement is hearsay not within any exception.BNo, because the statements of both the janitor and the supervisor are hearsay not within any exception.CYes, because the janitor's statement is a present sense impression, and the supervisor's statement is a statement of his then-existing state of mind.DYes, because the statements of both the janitor and the supervisor are statements concerning a matter within the scope of their employment.
(A) is correct. The janitor's statement to the plaintiff is hearsay because it was made out of court and is being offered for its truth-that the supervisor made a disparaging remark about the plaintiff and encouraged male co-workers to treat her unfairly. Because the janitor's statement does not fall within any hearsay exception, it is inadmissible. (B) is incorrect. A hearsay statement that incorporates other hearsay within it is known as "hearsay within hearsay" or "multiple hearsay." Hearsay within hearsay is admissible only if both the outer hearsay statement and the inner hearsay statement fall within an exception to the hearsay rule. Note that in the context of the hearsay within hearsay rule, a statement that is categorized as nonhearsay (e.g., an opposing party's statement) functions like a hearsay exception. Here, it is true that there are two out-of-court statements: (i) the supervisor's statement, and (ii) the janitor's statement repeating the supervisor's statement. It could be argued that the supervisor's instruction to the male co-workers was not an assertion and therefore not a "statement" for purposes of the hearsay rule. However, even if the supervisor's statement was an assertion being offered for its truth-that he wanted to drive the plaintiff out of the company-it would qualify as an opposing party's statement, which is not hearsay. Statements made by a party's employee concerning a matter within the scope of his employment, made during the existence of the employment relationship, are attributable to the party as a "vicarious" opposing party's statement. Here, the supervisor's instruction to the male co-workers was clearly made in the scope of his employment with the company. Thus, his statement is not hearsay when offered against the company. However, the janitor's statement to the plaintiff is inadmissible hearsay, and thus the entire testimony is inadmissible. (C) is incorrect. A present sense impression is a statement that describes or explains an event or condition and is made during or immediately after the declarant perceives the event or condition. Here, there is no indication that the janitor repeated the supervisor's statement to the plaintiff while he heard it or immediately after he heard it. Because the statement does not meet the timing requirement, it cannot be admitted as a present sense impression. Furthermore, the supervisor's statement is not hearsay because it is a statement attributable to an opposing party; it does not need to fall within a hearsay exception. Even if it were hearsay, the then-existing state of mind exception is probably not applicable because the supervisor did not directly assert his own emotion, intent, or motive, or otherwise describe his present state of mind. (D) is incorrect. Although the supervisor's statement to the co-workers was made within the scope of his managerial duties, the janitor's statement to the plaintiff had nothing to do with his job duties as a janitor. Therefore, the janitor's statement cannot be attributed to the company. Because the janitor's statement is hearsay and does not fall within any exception, the plaintiff's testimony is inadmissible.
A man purchased a house that needed substantial repairs. The man financed the purchase of the house by borrowing funds from a bank. He signed a note to the bank and secured repayment of the loan with a mortgage. After purchasing the house, the man borrowed money from his credit union to make the repairs. The man signed a note to the credit union; this note was also secured by a mortgage on the house. Both mortgages were promptly recorded in the order granted. The man was personally liable on both loans. The man moved into the house and made the necessary repairs. He later defaulted on the debt to the bank, and the bank initiated judicial foreclosure proceedings, naming both the man and the credit union as parties to the action. An outside party acquired the house at the foreclosure sale. After the expenses of the sale and the balance due the bank have been paid, $5,000 remains in the sale proceeds. The outstanding balance of the credit union loan is $20,000. The man and the credit union both claim the $5,000. There is no applicable statute. Who is entitled to the $5,000? Press Enter or Space to submit the answerAThe credit union, because the credit union has priority.BThe credit union, because the man is personally liable on the debt.CThe man, because of his equitable right of redemption.DThe man, because the outside party received title to the house subject to the second mortgage.
(A) is correct. The proceeds of the foreclosure sale are used first to pay expenses of the sale, attorneys' fees, and court costs; then to pay the principal and accrued interest on the loan that was foreclosed; next to pay off any junior liens or other junior interests in the order of their priority; and finally, any remaining proceeds are distributed to the mortgagor. Here, after the payment of expenses and costs and payment of monies due to the bank, $5,000 was remaining. The credit union, as a second mortgagee, is entitled to the money over the man. (B) is incorrect. If the proceeds of the foreclosure sale are insufficient to satisfy the mortgage debt, the mortgagee may be able to bring a personal action against the mortgagor/debtor for the deficiency. Thus, the man here is personally liable for the deficiency, but that is not the reason the credit union will recover the $5,000. It will recover because junior mortgagees are entitled to any foreclosure sale surplus after the senior mortgage is paid before any money is paid to the mortgagor. (C) is incorrect. The equitable right of redemption gives the mortgagor the right, at any time prior to the foreclosure sale, to redeem the land or free it of the mortgage by paying off the amount due, together with any accrued interest. Here, the man had the right to pay off the mortgage up to the time of the foreclosure sale. After the sale, he does not have the equitable right of redemption (although he may have a statutory right to redeem). Regardless, the right of redemption does not entitle him to the $5,000. (D) is incorrect. When a mortgage is foreclosed, the buyer at the sale will take title as it existed when the mortgage was placed on the property. Thus, foreclosure will terminate interests junior to the mortgage being foreclosed but will not affect senior interests. Here the mortgage that was foreclosed was the first mortgage, and thus the credit union's junior mortgage is terminated. The outside party takes the property free of any mortgage.
Fifteen years ago, two men who were fishing buddies moved onto vacant rural land owned by a woman they didn't know and built a small fishing shack on it. Twelve years ago, the men replaced the shack with a fish processing plant and a commercial fishing boat dock. The men maintained their commercial fishery operation on the land until one of them died intestate last year, leaving a sole heir. The period of time to acquire title by adverse possession in the jurisdiction is 10 years. The woman has now become aware of the changes that have occurred on the land. In an action to determine title, for whom should the court decide? Press Enter or Space to submit the answerAFor the man who is still alive and the deceased man's heir, because the men acquired title as tenants in common.BFor the man who is still alive, because he is the surviving adverse possessor.CFor the woman, because the use was changed by the men while they were in possession.DFor the woman, because title cannot be claimed by two adverse possessors simultaneously.
(A) is correct. Title to the property was acquired by adverse possession. For adverse possession, the possession must be: (i) actual (it must give the true owner notice of the trespass) and exclusive (not shared with the true owner or the public); (ii) open and notorious possession (use of the property as an owner would be sufficient to put the true owner on notice of the trespass); (iii) adverse (sometimes called hostile, which means without the owner's permission); and (iv) continuous for the statutory period. Two or more people can work together to take title by adverse possession, and if they meet the requirements, they take as tenants in common. Here, the men took actual possession of the land by building a fish processing plant and dock. They did not share possession with the woman or the public. Their use of the property was as an owner might use the land, and the improvements and activities on the land were sufficient to put the woman on notice. The men did not have the woman's permission, and possession was continuous for more than is required by the statute. Therefore, they gained title by adverse possession and took the property as tenants in common. A tenancy in common is a concurrent estate with no right of survivorship. Each tenant has a distinct, undivided interest in the property. This interest is freely alienable and is inheritable. When one of the men died intestate, leaving a sole heir, the heir inherited the man's interest in the property. Note that building the fishing shack may not have been enough for adverse possession. It was small and possibly not enough to give notice to the woman of the trespass. It was probably used only occasionally for fishing, and that may be a use that an owner of the property would make, but that is not certain. In any case, the plant and dock were open and notorious and for the statutory period. (B) is incorrect. When two people work together to take title by adverse possession, they take as tenants in common, an estate that has no right of survivorship. Thus, the man who is still alive is a tenant in common with the deceased man's heir. (C) is incorrect. Change of use is not relevant as long as each use meets the requirements for adverse possession. In any case, the use of the processing plant and dock was for more than the statutory period, so the men would take by adverse possession based on that one use. (D) is incorrect. Two or more people may work together to take by adverse possession.
A seller contracted to sell land to a buyer for $300,000.The contract provided that the closing would be 60 days after the contract was signed and that the seller would convey to the buyer a "marketable title" by a quitclaim deed at closing. The contract contained no other provisions regarding the title to be delivered to the buyer. A title search revealed that the land was subject to an unsatisfied $50,000 mortgage and a right-of-way easement over a portion of the land. The buyer now claims that the title is unmarketable and has refused to close. Is the buyer correct? Press Enter or Space to submit the answerANo, because nothing under these facts renders title unmarketable.BNo, because the buyer agreed to accept a quitclaim deed.CYes, because the right-of-way easement makes the title unmarketable.DYes, because the unsatisfied mortgage makes the title unmarketable.
(C) is correct. An easement that reduces the value of the property, such as an easement of way for the benefit of a neighbor, renders title unmarketable. Thus, (A) is incorrect. (B) is incorrect because the fact that a contract calls for a quitclaim deed, which makes no warranties for title, does not affect the implied covenant to provide marketable title. The type of deed affects remedies available after closing, not before. (D) is incorrect because, even though a mortgage can render title unmarketable, the seller has the right to satisfy the mortgage at closing with the proceeds of the sale. Thus, if the price is sufficient, the closing will result in marketable title.
In a tavern, an intoxicated woman threatened to slash a man with a broken beer bottle. Another customer, who had not been threatened by the woman, forcefully grabbed the woman and locked her in the tavern's storeroom until the police could arrive. In the process, although the customer used reasonable force, the customer badly sprained the woman's wrist. Is the woman likely to recover in an action against the customer? Press Enter or Space to submit the answerANo, because the customer's conduct was privileged as a defense of others.BYes, based on battery only.CYes, based on false imprisonment only.DYes, based on both battery and false imprisonment.
(A) is correct. When a person has reasonable grounds to believe that a third person is being, or is about to be, attacked, he may use such force as is reasonably necessary to protect the third person against the potential injury. Here, the customer intervened to prevent a man from being slashed with a broken beer bottle, and used only reasonable force against the woman. Hence, the customer's conduct was privileged as a defense of others. (B) is incorrect. Although the customer intentionally used harmful force against the woman, his conduct was privileged as a defense of others. Although he badly sprained the woman's wrist, he used only reasonable force to prevent her from slashing someone with the broken beer bottle. (C) is incorrect. Although the customer intentionally confined the woman to a bounded area, his conduct was privileged as a defense of others. He locked the woman in the storeroom just until the police could arrive so that she would not try to harm the man or anyone else in the tavern. (D) is incorrect. Although the customer intentionally used harmful force against the woman and confined her to a bounded area, his conduct was privileged as a defense of others.
A man conveyed the eastern half of a tract of vacant land to a woman by a warranty deed. The woman promptly recorded the deed. The land conveyed to the woman fronted on a public highway. The land retained by the man was landlocked. One year later, the man died intestate, leaving a cousin as his only heir. The cousin visited the man's land for the first time and discovered that it had no access to a public highway. A neighbor who owned adjoining land fronting on the public highway offered to sell the cousin a right to cross the neighbor's land for access to the highway. Although the neighbor's price was reasonable, the cousin rejected the offer. The woman has refused to allow the cousin to cross her land for access to the public highway even though the woman's land is still vacant. The cousin has sued the woman, seeking access across the woman's land to the public highway. Who is likely to prevail? Press Enter or Space to submit the answerAThe cousin, based on necessity.BThe cousin, because the woman's land is still vacant.CThe woman, because the cousin could obtain an alternative access to the highway from the neighbor.DThe woman, because the man failed to reserve an easement in his deed to the woman.
(A) is correct. When the owner of a tract of land sells a part of it and by this division deprives one lot of access to a public road, a right-of-way by absolute necessity is created by implied grant or reservation over the lot with access to the public road. The owner of the servient parcel has the right to locate the easement, provided the location is reasonably convenient. Here the man owned the original tract and sold a part of it to the woman. When he sold the part, his remaining tract was landlocked. Therefore, a right-of-way by absolute necessity was created to give him access to the public road. The woman can decide where to locate the right-of-way, but she cannot deny it altogether. This easement by necessity is an easement appurtenant, meaning that the right of special use benefits the holder of the easement in his physical use or enjoyment of another tract of land. The benefit of an easement appurtenant becomes an incident of the possession of the dominant tenement. All who possess or subsequently succeed to title to the dominant tenement become, by virtue of the fact of possession, entitled to the benefit of the easement. Therefore, the cousin who inherited the land is entitled to use the right-of-way. (B) is incorrect. The cousin, as the successor to the landlocked parcel, is entitled to an easement by necessity. The fact that the woman's land is vacant has no bearing on whether or not the cousin has a right to cross her property. (C) is incorrect. When the owner of a tract of land sells a part of it and by this division deprives one lot of access to a public road, a right-of-way by absolute necessity is created over the lot with access to the public road. This is true regardless of whether the landlocked owner could obtain a right-of-way from another neighbor. (D) is incorrect. The easement here did not have to be reserved in the deed; an easement by necessity is an implied easement. It was created when the man sold the woman a portion of his property resulting in his tract becoming landlocked.
A homeowner resides downhill from a metal fabrication facility. She has sued both the owner of the facility and the supplier of a solvent used at the facility. She contends that contaminants, consisting mostly of the solvent, were released into the ground at the facility and have migrated and continue to migrate to her property, contaminating the soil, the groundwater, and her well. She alleges various acts of negligence on the part of the facility owner in causing the release of the contaminants into the ground. She also alleges that employees of the solvent supplier were negligent in frequently spilling some of the solvent onto the ground while filling a rooftop tank at the facility. The solvent supplier has moved for summary judgment, arguing that if there was any contamination, the facility owner and the supplier independently contributed indeterminate amounts to the contamination and that therefore the homeowner cannot show how much damage each has inflicted on her. There is no evidence that the facility owner and the solvent supplier acted in concert. Should the court grant the summary judgment motion? Press Enter or Space to submit the answerANo, because concurrent tortfeasors are jointly and severally liable for an indivisible injury.BNo, because the solvent supplier is vicariously liable for damage inflicted by the facility owner.CYes, because there is no basis for allocating damages against the solvent supplier.DYes, because there is no evidence that the facility owner and the solvent supplier acted in concert.
(A) is correct. When two or more tortious acts combine to proximately cause an indivisible injury to a plaintiff, each tortfeasor is jointly and severally liable for that injury. This means that each is liable to the plaintiff for the entire damage incurred. Joint and several liability applies even though each tortfeasor acted entirely independently. Hence, the homeowner need not show how much damage each tortfeasor has inflicted on her; each of them may be liable to her for the entire amount of the damage. Accordingly, the summary judgment motion should be denied. (B) is incorrect. The solvent supplier and the facility owner are in a contractual relationship; there is no basis for imposing vicarious liability on the solvent supplier for the damage inflicted by the facility owner. (C) is incorrect. As discussed above, joint tortfeasors are each liable to the plaintiff for the entire damage incurred. The plaintiff need not present a basis for allocating damages between the tortfeasors. (D) is incorrect. When two or more tortfeasors act in concert and injure the plaintiff, each will be jointly and severally liable even if the injury is divisible and it could be determined what each torfeasor had done. However, joint and several liability also is available where the tortfeasors both contributed to an indivisible injury, as discussed above.
A patient domiciled in State A sued a surgeon domiciled in State B in a federal court in State A, alleging claims for malpractice. The surgeon moved to dismiss the action for lack of personal jurisdiction. The court denied the motion and set discovery cutoff and trial dates. The surgeon has appealed the denial of the motion. Should the appellate court hear the merits of the surgeon's appeal? Press Enter or Space to submit the answerANo, because the appellate court lacks jurisdiction over the appeal.BNo, because the district court's decision on jurisdiction is final.CYes, because a contrary appellate decision could terminate the action.DYes, because the surgeon's personal-jurisdiction challenge raises a constitutional question.
(A) is correct. With certain exceptions by rule or statute, only final judgments may be appealed. A final judgment is one that disposes of the whole case on its merits. Since the case was still pending after the denial of a motion to dismiss on the basis of a lack of personal jurisdiction, the judgment here was not a final judgment and thus was not appealable. (B) is incorrect because an order determining personal jurisdiction may be appealed (and is not within the sole discretion of the district court), but the order must be "final" (that is, it must dispose of all claims by all parties). (C) is an incorrect statement of law and the final order rule. The order must be "final" at the trial court level, not be potentially made final by an appellate court ruling. (D) again is an incorrect statement of law. The final order rule may not be circumvented by the mere raising of a constitutional issue.
A man filed a federal diversity action against a bus company, seeking damages for injuries he had sustained in an accident while riding a bus owned by the company. The man demanded a jury trial. After the parties' attorneys examined the prospective jurors and exercised their challenges, six jurors and two alternate jurors were chosen. During the trial, two jurors became ill and were replaced by the alternate jurors. At the conclusion of the trial, a third juror also became ill, and the court excused that juror. The parties' attorneys stipulated to the return of a verdict from a five-person jury. The jury then deliberated and returned a verdict for the company. The man timely filed a motion for a new trial, arguing that the five-person jury was not large enough to return a verdict. Should the court grant the motion? Press Enter or Space to submit the answerANo, because the court properly excused the three jurors due to illness.CorrectBNo, because the parties stipulated to a verdict from a jury of fewer than six jurors.IncorrectCYes, because there must be at least six jurors on a federal civil jury.DYes, because there must be at least 12 jurors on a federal civil jury.
(B) is correct. A jury must be composed of at least six jurors at the beginning of the trial. If the number drops below six (for example, as here, when jurors became ill), a mistrial results, unless the parties agree to a lesser number of jurors. Because the parties stipulated to fewer jurors, the issue is waived. (A) is incorrect because even though the jurors were properly excused, a lesser number of jurors cannot return a verdict unless the parties agree. (C) is incorrect because the parties may stipulate to fewer jurors. (D) is incorrect. A jury must be composed of at least six jurors, not 12, and the answer does not address the possibility of a stipulation.
A man was in jail after being arrested for burglary. When the police attempted to question him, the man invoked his Miranda rights and refused to answer any questions. The man was subsequently tried, convicted, and sentenced to a prison term for the burglary. Three years later, while the man was serving his prison sentence for the burglary, a police detective from a nearby town questioned him about an unsolved homicide. The detective did not know that the man had invoked Miranda at an earlier time. The man waived his Miranda rights and made several incriminating statements to the detective. When he was later charged with the homicide, the man moved to suppress these statements, claiming that his earlier refusal to waive his Miranda rights should have been honored. Should the court suppress the statements? Press Enter or Space to submit the answerANo, because the detective was unaware that the man had originally invoked his Miranda rights.BNo, because the man's prior invocation of his Miranda rights did not preclude the later interrogation.CYes, because the man had earlier invoked his Miranda rights, and the police were not permitted to resume questioning, even after a time lapse of years.DYes, because the man was incarcerated, and his earlier invocation of his Miranda rights shielded him from further questioning until he was released.
(B) is correct. As a general rule, police officers must scrupulously honor a detainee's invocation of Miranda rights. If the invocation is not scrupulously honored, statements made by the detainee are inadmissible. The prohibition against questioning a detainee after invoking Miranda rights lasts the entire time the detainee is in custody for interrogation purposes, plus 14 more days after the detainee returns to his normal life (which can include his "normal life" in jail). Here, the detainee invoked Miranda rights three years before the questioning in the current case. Thus, far more than 14 days had passed since he had returned to his "normal life" in jail. Therefore, (B) is correct and (C) and (D) are incorrect. (A) is incorrect because whether the detective knew the man had originally invoked his Miranda rights is irrelevant: his incarceration and time lapse of three years rendered the prior invocation of his rights moot. The man would need to be given new Miranda warnings regardless.
A plaintiff sued a department store for injuries she sustained when she slipped and fell in the store. At trial, the plaintiff proposes to testify that, when the store manager rushed to the scene, he said to the plaintiff, "I'm so sorry about the water on the floor there, but don't worry-the store will pay for the ambulance and your hospital bill." The store's attorney objects. How should the court rule on the admissibility of the store manager's testimony? Press Enter or Space to submit the answerAThe testimony is admissible in its entirety as the statement of an opposing party.BThe testimony about the water is an admissible statement of an opposing party, but the rest of the testimony is inadmissible as an offer to pay medical expenses.CThe testimony is inadmissible in its entirety, because it is hearsay not within any exception.DThe testimony is inadmissible in its entirety, because the manager's statement is in the context of an offer to pay medical expenses.
(B) is correct. Evidence that a party paid, or offered to pay, an injured party's medical and similar expenses is not admissible to prove liability for the injury. This public policy exclusion is based upon the concern that such payment might be prompted solely by humanitarian motives. However, only the offer itself is excluded under this rule; accompanying statements (such as admissions of fact) may be admitted. Here, the manager's offer to pay the ambulance and hospital bills will clearly be excluded. However, the manager's accompanying statement about the water is admissible, even though it appears to meet the general definition of hearsay. While it was made out of court and is being offered for its truth-that there was water on the floor-it is a statement of an opposing party, which is not hearsay. A statement made by a party's employee concerning a matter within the scope of employment, and made during the existence of the employment relationship, is attributable to the party. The manager's statement about the condition of the floor was clearly within the scope of his employment, and thus his statement is admissible against the store. (A) is incorrect. The testimony is not admissible in its entirety; the offer to pay the ambulance and medical bills will be excluded as a matter of public policy. (C) is incorrect. The manager's statement is admissible against the store as an opposing party's statement because it was made in the scope of the manager's employment with the store and during the existence of the employment relationship. An opposing party's statement is not hearsay. (D) is incorrect because the rule excluding offers to pay medical expenses only applies to the offer or payment itself. Accompanying statements, such as the manager's statement about the water, may be admitted. (Contrast the public policy exclusion for compromise negotiations, which applies to both the offer itself and any other statements made in the negotiation.)
A man who believed that his wife was cheating on him with her gym trainer decided to kill the trainer. He loaded his handgun and set off for the trainer's house. Because he was anxious about committing the crime, the man first stopped at a bar, drank eight shots of hard liquor, and became intoxicated. He then left the bar and went to the trainer's house. When the trainer answered the door, the man shot and killed him. The man then passed out on the trainer's porch. The man has been charged with murder in a jurisdiction that follows the common law. Can the man raise an intoxication defense? Press Enter or Space to submit the answerANo, because drinking at the bar was the proximate cause of the killing.BNo, because the man intended to commit the murder and drank to strengthen his nerve.CYes, because drinking at the bar was a foreseeable intervening cause of the killing.DYes, because the man's intoxication negated the specific intent required for murder.
(B) is correct. Murder is the unlawful killing of a human being with malice aforethought. Malice aforethought exists if the defendant has any of the following states of mind: (i) the intent to kill, (ii) the intent to inflict great bodily injury, (iii) reckless indifference to an unjustifiably high risk to human life, or (iv) the intent to commit a felony. Voluntary intoxication may be a defense to specific intent crimes if it prevented the defendant from formulating the requisite intent. Here, the man had the specific intent to kill before he drank any alcohol. He decided to kill the trainer, loaded his handgun, and set off for the trainer's house before going to the bar. The man's intoxication did not prevent him from forming the intent to kill; it reduced his anxiety so he could continue with his plan. He therefore cannot raise an intoxication defense. (A) is incorrect. To be guilty of murder, a defendant's conduct must be both the cause-in-fact and the proximate cause of the death. A defendant's acts are the proximate cause of all results that occur as a natural and probable consequence of his conduct, and the chain of proximate causation can be broken only by the intervention of a superseding factor. Here, the man intended to kill the trainer before he drank at the bar. His drinking therefore did not cause him to have the intent to kill and was not the proximate cause of the killing. (C) is incorrect. An intervening act will shield a defendant from liability if the act is a mere coincidence or is outside the foreseeable sphere of risk created by the defendant's act. As discussed above, the man had the intent to kill before he began drinking. The drinking was not a mere coincidence or unforeseeable; the man purposely drank at the bar to ease his anxiety so he could go through with the killing. (D) is incorrect. Voluntary intoxication is a defense to a specific intent crime only if it prevented the defendant from formulating the requisite intent. As described above, the man had the specific intent to kill before he drank at the bar, so his intoxication did not prevent him from forming the intent.
Two days before his home was to be sold at a foreclosure sale, a homeowner obtained a temporary restraining order (TRO) in federal court that prevented his lender from proceeding with the sale for 14 days or until a preliminary injunction hearing could take place, whichever was sooner. When a preliminary injunction hearing could not be scheduled within the original 14-day period, the court extended the TRO for another 30 days. The lender appealed the court's order extending the TRO. The homeowner has moved to dismiss the appeal. Is the appellate court likely to dismiss the appeal? Press Enter or Space to submit the answerANo, because a TRO is immediately appealable.BNo, because the 30-day extension makes the TRO equivalent to a preliminary injunction and therefore appealable.CYes, because a TRO is not appealable under the interlocutory appeals statute.DYes, because there is no final judgment from which an appeal may be taken.
(B) is correct. Ordinarily, the granting or denying of a temporary restraining order is not immediately appealable because, as the name implies, a temporary restraining orderis not an injunction that is appealable under 28 U.S.C. section 1292(a)(1). A TRO is valid for 14 days, unless the TRO is extended for good cause for a like period. If an extension goes beyond the "like period," courts have held that the TRO is effectively converted into a preliminary injunction and thus becomes immediately appealable under the statute. (A) is incorrect because, as stated, a TRO is not immediately appealable. (C) is incorrect. The label placed on the order/injunction is of less importance than its effect. If a party is restrained beyond the applicable time periods for a TRO, the order is a preliminary injunction regardless of what label is used. (D) is incorrect. Interlocutory injunctions are appealable under the statute despite the fact that the order is not final.
A city filed eminent domain proceedings in order to obtain 40 beach houses fronting a particularly attractive stretch of shoreline. As part of an elaborate plan to increase the city's tourist trade and revive the local economy, the city planned to sell the beach houses to a company that would demolish the houses and build a luxury hotel in their place. The owners of the beach houses have challenged the city's exercise of eminent domain, contending only that the city's plan is unconstitutional. Will the owners of the beach houses be likely to prevail? Press Enter or Space to submit the answerANo, because a property owner can challenge an exercise of eminent domain only on the ground of the sufficiency of the compensation.BNo, because the planned sale to the private developer to increase the tourist trade qualifies as a public use.CYes, because a public entity cannot seize the property of one person in order to transfer that property intact to other private parties.DYes, because the city's action would deprive the owners of all economic use of their property.
(B) is correct. The Fifth Amendment prohibits taking of private property for public use without just compensation. The Supreme Court has held that taking property to turn it over to a private developer can still qualify as a public use. (A) is incorrect because it is not true. The Fifth Amendment requires both just compensation and a public use as prerequisites to exercise of the power of eminent domain. (C) is incorrect because, as indicated above, it is not true. The Supreme Court has approved such transfers. So long as it appears it was reasonable for the government to believe the transfer to the private entity will benefit the public, such as by promoting economic development, the public use requirement is met. (D) is incorrect for the reasons stated above.
A state law provides for an award of damages against anyone who publishes the name of a rape victim. Pursuant to that law, a woman sued a local newspaper in state court after the newspaper identified her as a rape victim. The state trial and appellate courts rejected the claim, holding that the state law was invalid under both the state constitution and the First Amendment of the U.S. Constitution. The state supreme court affirmed, holding specifically: "We think that this well-intentioned law very likely violates the First Amendment of the federal Constitution. We need not, however, decide that issue, because the law assuredly violates our state constitution, which provides even greater protection to the right of the press to report the news." The woman petitioned for review in the U.S. Supreme Court. Is the U.S. Supreme Court likely to review the state supreme court judgment? Press Enter or Space to submit the answerANo, because the First Amendment prohibits the imposition of liability for the publication of truthful information.BNo, because the judgment of the state supreme court rests upon an adequate and independent state-law ground.CYes, because the Supremacy Clause does not permit a state to create rights greater than those conferred by the federal Constitution.DYes, because the U.S. Supreme Court's appellate jurisdiction extends to cases arising under federal law.
(B) is correct. The Supreme Court will hear a case from a state court only if the case turns on federal grounds. The Court will not hear a case if it finds adequate and independent nonfederal grounds to support the state decision. A ground is adequate if it is completely dispositive of the case, and it is independent if it does not rely on federal law. Here, the state court held that its decision rested solely on interpretation of its own constitution, and the state court decision completely disposed of the matter. Thus, it would be a waste for the Supreme Court to hear the case, because even if it decided the law was constitutional under the First Amendment, the law would remain unconstitutional under the state constitution. (A) is incorrect both because it is overbroad (for example, truthful publication of material in violation of another's copyright can be punished) and because the Court will not waste its time on a case where its decision will not change a result because the case rests on adequate and independent nonfederal grounds. (C) is simply not true. The Supremacy Clause does not prevent states from creating rights greater than those conferred by the federal Constitution - the federal Constitution is a floor and not a ceiling. It provides the minimal level of protection. States are free to provide greater protection. (D) is true but irrelevant. The Supreme Court does have appellate jurisdiction to hear cases arising under federal law, but the Court does not have to hear every case. Under the doctrine of strict necessity, it will not hear a case like this one, that rests on adequate and independent, nonfederal grounds.
A general contractor about to bid on a construction job for an office building invited a carpenter and several others to bid on the carpentry work. The carpenter agreed to bid if the general contractor would agree to give the carpenter the job provided that his bid was lowest and the general contractor was awarded the main contract. The general contractor so agreed. The carpenter, incurring time and expense in preparing his bid, submitted the lowest carpentry bid. The general contractor used the carpenter's bid in calculating its own bid, which was successful. Which of the following best supports the carpenter's position that the general contractor is obligated to award the carpentry subcontract to the carpenter? Press Enter or Space to submit the answerAThe carpenter detrimentally relied on the general contractor's conditional promise in preparing his bid.BThe carpenter gave consideration for the general contractor's conditional promise to award the carpentry subcontract to the carpenter.CThe general contractor has an implied duty to deal fairly and in good faith with all bidders whose bids the general contractor used in calculating its main bid to the building owner.DThe general contractor has an obligation to the owner of the building to subcontract with the carpenter because the carpenter's bid was used in calculating the general contractor's bid, and the carpenter is an intended beneficiary of that obligation.
(B) is correct. The carpenter's bid was consideration for the general contractor's promise to award the carpentry subcontract to the carpenter if his bid was the lowest and the general contractor was awarded the main contract. Thus, the general contractor and the carpenter formed a contract. Two elements are necessary to constitute consideration. First, there must be a bargained-for exchange between the parties; and second, that which is bargained for must constitute a benefit to the promisor or a detriment to the promisee. The carpenter and the general contractor agreed that the carpenter would supply a bid that the general contractor could use in its own bid. The carpenter's bid was bargained for and was a benefit to the general contractor, so it constitutes consideration sufficient to support the general contractor's conditional promise to award the subcontract to the carpenter. Conditional promises are enforceable, but the duty to perform does not become absolute until the condition has been met or is legally excused. The conditions in this contract were met-the carpenter's bid was the lowest and the general contractor was awarded the main contract. Thus, the general contractor is under a duty to perform his promise to award the subcontract to the carpenter. (A) is not the best answer. The carpenter is seeking specific performance of their agreement so that he will be awarded the carpentry subcontract. If the carpenter uses a detrimental reliance or promissory estoppel argument, he would be conceding that he gave no consideration and there is no contractual obligation, but he should be awarded damages to prevent injustice. Courts will often limit damages under this theory to reliance damages, which could be much less than the value of the subcontract. (C) is incorrect. Although a general contractor should deal with all bidders in good faith, this is not a reason why this particular carpenter would be awarded the subcontract. The carpenter should be awarded the subcontract because the general contractor gave him a conditional promise that he would do so in exchange for his bid. (D) is incorrect. A bid is not a third party beneficiary contract. Generally speaking, a contractor has no obligation to award a subcontract to a particular subcontractor unless there was a separate agreement that it would do so, such as the case here where the general contractor gave the carpenter a conditional promise that it would award the carpenter the subcontract if his bid was the lowest and if the general contractor was awarded the general contract.
A statute passed by both houses of Congress and signed by the President authorizes a federal agency to select a site for and to construct a monument honoring members of the capitol police force killed in the line of duty. The statute appropriates the necessary funds but provides that the funds may not be expended until both houses of Congress have adopted a concurrent resolution, not subject to presentment to the President, approving the agency's plans for the monument's location and design. Is the provision requiring further congressional approval before expenditure of the funds constitutional? A No, because decisions regarding the placement and design of government-owned structures are an exclusively executive function with which Congress may not interfere by any means. B No, because the provision amounts to an unconstitutional legislative interference with an executive function. C Yes, because Congress may attach reasonable conditions to its appropriation of funds to executive departments, and its special interest in the members of its own police force makes the provision a reasonable condition. D Yes, because the provision is part of a statute that was passed by both houses of Congress and signed by the President.
(B) is correct. The enactment of laws by Congress requires passage of the law in both houses (bicameralism) and approval of the law by the President (that is, the presentment requirement) or an override of a presidential veto. Here, Congress passed a law and the President signed it, but Congress sought further control by requiring expenditures to be approved specifically by Congress without presentment to the President. Such a requirement usurps the power of the executive branch to execute laws and places it in the hands of Congress in violation of the doctrine of separation of powers. (A) is incorrect. Under the Spending Clause, Congress certainly has the power to adopt a law specifying where a monument should be placed, and if such a law were passed by Congress and the President signed the law, it would be valid. However, that is not what happened here. Here, the law left it to an agency to select a site. (C) is incorrect for the reasons stated above; requiring subsequent Congressional approval of agency site selection is unconstitutional. (D) is incorrect for the same reason.
An engineer signed a two-year contract to serve as the chief safety engineer for a coal mine at a salary of $7,000 per month. The position required the engineer to work underground each workday. After one week on the job, the engineer became very ill. He requested testing of the mine air system, which revealed the presence of a chemical agent to which the engineer had a rare allergic reaction. The engineer promptly quit. The coal mine then hired a qualified replacement for the remainder of the engineer's contract at a salary of $7,500 per month. Assume that no statute or regulation applies. If the coal mine sues the engineer for breach of contract, is the coal mine likely to recover damages? Press Enter or Space to submit the answerANo, because an at-will employee has the right to terminate an employment contract.BNo, because the risk to the engineer's health excused his nonperformance of the contract.CYes, because the coal mine acted in good faith in mitigating the effect of the engineer's failure to finish the contract term.DYes, because the mine is reasonably safe for most people.
(B) is correct. The engineer's nonperformance will be excused by impracticability. The occurrence of an unanticipated or extraordinary event may make contractual duties impossible or impracticable to perform. Where the nonoccurrence of the event was a basic assumption of the parties in making the contract and neither party has expressly or impliedly assumed the risk of the event occurring, contractual duties may be discharged. The test for a finding of impracticability is that the party to perform has encountered: (i) extreme and unreasonable difficulty and/or expense; and (ii) its nonoccurrence was a basic assumption of the parties. The engineer's rare allergic reaction to a chemical found in the mine air system was unanticipated by the parties. The engineer cannot perform his job anywhere but inside the coal mine but it is unreasonable for the engineer to risk his health, and possibly his life, to fulfill his duties under the contract. Due to this unanticipated and rare circumstance, his employment under the contract will be discharged. (A) is incorrect. All employees are free to terminate their employment because involuntary servitude is prohibited by the Constitution, but in the case of an employee under a contract, doing so is a breach of contract. If the engineer's performance was not excused by impracticability, then the coal mine could have recovered damages for breach. (C) is incorrect. If the engineer's performance had not been excused, then the mitigation by the coal mine would have been relevant to damages. However, due to the unanticipated extraordinary circumstances of the engineer's allergic reaction, the contract was discharged. (D) is incorrect. It does not matter that the mine was reasonably safe for most people; the test for impracticability is a subjective one. The engineer could not perform under the contract without risking illness and possibly death due to his unusual and unanticipated allergy. Thus his performance under the contract will be discharged due to impracticability.
A firm contracted with a municipality to repair a drawbridge and began work on February 1. The contract provided that the firm would be paid an additional $1,000 for each day the repair was completed before the April 1 completion deadline. On March 14, an unusual gear on the bridge broke. On that same day, the firm contracted with a supplier for a March 15 delivery to the bridge site of the only available replacement gear.The supplier did not know about the early-completion incentive-pay provision in the firm's contract. The supplier misdirected the delivery of the gear, and the firm did not receive it until March 20. The work on the bridge was completed on March 21. But for the late delivery of the gear, the firm would have completed the bridge repair on March 16. In an action against the supplier for breach of contract, will the firm be permitted to recover, as part of its damages, the additional $5,000 early-completion incentive pay it would have received from the municipality but for the supplier's breach? Press Enter or Space to submit the answerANo, because the supplier did not expressly agree to be responsible for such damages.BNo, because the supplier did not know and had no reason to know about the early-completion incentive-pay provision.CYes, because the supplier is liable for all damages flowing directly from its breach.DYes, because, by the nature of its contract with the firm, the supplier should have known that time was of the essence.
(B) is correct. The firm cannot recover the $5,000 early-completion pay as consequential damages. Consequential damages are special damages and reflect losses over and above standard expectation damages. These damages result from the nonbreaching party's particular circumstances. Consequential damages may be recovered only if at the time the contract was made, a reasonable person would have foreseen the damages as a probable result of a breach. To recover consequential damages, the plaintiff must show that the breaching party knew or had reason to know of the special circumstances giving rise to the damages. Here the supplier did not know, or have reason to know, about the early-completion incentive-pay provision in the firm's contract, so consequential damages for those losses cannot be recovered. (A) is incorrect. A party does not have to expressly agree to be responsible for consequential damages. If the party knew or had reason to know such damages would be a probable result of a breach, that party can be held responsible for consequential damages. (C) is incorrect because it is overbroad. Again, a party to a contract is not liable for all damages that might result from breach. A party is responsible for consequential damages that reflect losses over and above standard expectation damages only if the party would have foreseen the damages as a probable result of a breach. (D) is also incorrect. Even though the contract called for next day delivery, nothing in the contract indicated time was of the essence. Moreover, even if we assume that time was of the essence, that does nothing to give the supplier notice that delay would cost the firm $1,000 per day. Thus, the delay damages were still not foreseeable. This was a standard order. No information was given to the supplier to alert the supplier that any delay would cause such extensive damage to the firm.
A woman owned land in fee simple absolute. The woman conveyed the land to a friend "for life," and when the friend died the land was to go to the woman's neighbor "and her heirs." The neighbor died and in her duly probated will devised her entire estate to a local charity. If she had died intestate, her daughter would have been her only heir. One year after the neighbor died, her daughter executed a quitclaim deed conveying any interest she might have in the land to the woman's friend. The common law Rule Against Perpetuities is unmodified in the jurisdiction. There are no other applicable statutes. Who has what interest in the land? Press Enter or Space to submit the answerAThe friend has a fee simple absolute, because his life estate merged with the remainder conveyed to him by the daughter.BThe friend has a life estate and the charity has a vested remainder, because the neighbor's interest was devisable.CThe friend has a life estate and the daughter has a vested remainder, because the deed from the woman created an interest in the neighbor's heirs.DThe friend has a life estate and the woman has a reversion, because the neighbor's remainder was void under the Rule Against Perpetuities.
(B) is correct. The grant here gave a life estate to the friend and a remainder in fee simple to the neighbor. It is a remainder because on the friend's death (the natural termination of the preceding estate), the neighbor has the right to possession. The remainder is vested because it was granted to an ascertainable person in being (the neighbor) and there were no conditions to prevent it from becoming possessory. A vested remainder is devisable by will; so when the neighbor devised her estate to the local charity, the charity took her vested remainder interest. (A) is incorrect. Whenever the same person acquires all of the existing interests in land, present and future, a merger occurs, and that person then holds a fee simple absolute. Here, however, the daughter had nothing to convey to the friend. The woman conveyed a vested remainder in fee simple to the neighbor. That vested remainder is devisable, and the neighbor properly devised it by will to the local charity. The daughter, as an intestate heir, would have taken no interest in the property. A quitclaim deed conveys only the interest the grantor has, and in this case, that was nothing. Therefore, there was no merger. The daughter's deed conveyed no interest to the friend, and the friend has only his life estate. (C) is incorrect. The phrase in the grant "and her heirs" indicates that a fee simple interest was being granted; it does not give the heirs any rights in the property. Thus, as discussed, the daughter acquired no interest in the remainder. (D) is incorrect. The Rule Against Perpetuities states that no interest in property is valid unless it must vest, if at all, within 21 years after a life in being at the creation of the interest. Here, the neighbor's interest is already vested on creation; she has a vested remainder. Her estate is a vested remainder because it followed the natural termination of the preceding estate, and it was granted to an ascertainable person in being without any conditions to prevent it from becoming possessory. Because it is a vested remainder, it is valid under the Rule.
On the basis of scientific studies showing a causal relationship between the consumption of "red meat" (principally beef) and certain forms of cancer, a federal statute prohibits all commercial advertising of red meat products. The statute does not, however, restrict the sale of red meat products. Producers of red meat have challenged the statute as a violation of their free speech rights protected by the First Amendment. Is the court likely to find the statute constitutional? Press Enter or Space to submit the answerANo, because it does not serve a substantial government interest.BNo, because it is more extensive than necessary to serve the government interest in preventing certain cancers.CYes, because it does not affect speech protected by the First Amendment.DYes, because it serves a legitimate government interest in protecting public health.
(B) is correct. The regulation here involves commercial speech. Commercial speech is protected by the First Amendment, but the Court tests regulation of commercial speech under a special test. The Court first asks whether the speech is about a lawful activity and is truthful and not misleading. If these conditions are not satisfied, the speech has no protection. If they are satisfied, the regulation will be valid only if it (1) serves a substantial government interest, (2) directly advances that interest, and (3) is narrowly tailored to achieve that interest (that is, there is a reasonable fit between the means chosen and the ends sought). Here, the sale of red meat is allowed and so the producers are looking to advertise about a lawful activity. The regulation bans all commercial advertising. While the regulation serves a substantial government interest (that is, discouraging the consumption of a product linked to cancer), the regulation imposes a complete ban on advertisement. A complete ban will never be found to be narrowly tailored. (A) is incorrect. The government will no doubt be found to have a substantial interest in preventing cancer. (C) is incorrect. As indicated above, commercial speech is protected by the First Amendment. (D) is incorrect. First, it is not enough merely that the interest served is legitimate - it must be substantial. And second, even if a legitimate interest were enough, that is only one prong of the test, and the regulation here would still fail because it is not narrowly tailored.
A 15-year-old boy was killed during a gang fight. Two days after his funeral, the boy's mother saw a television program about gang violence and was shocked to see video of herself weeping over the boy's body. The video had been shot by the television reporting team while the boy's body was still lying on a public street. The mother suffered severe emotional distress as a result of seeing the video. If the mother sues the television station for invasion of her privacy and that of her son, will the mother be likely to prevail? Press Enter or Space to submit the answerANo, because a person has no right to privacy after his or her death.CorrectBNo, because the street was open to the public and the subject was newsworthy.CYes, because the mother did not give permission to have the video used in the program.DYes, because the mother suffered severe emotional distress as a result of viewing the video.
(B) is correct. The tort of invasion of privacy includes the following four kinds of wrongs: (i) appropriation by the defendant of the plaintiff's picture or name for the defendant's commercial advantage; (ii) intrusion by the defendant on the plaintiff's affairs or seclusion; (iii) publication by the defendant of facts placing the plaintiff in a false light; and (iv) public disclosures of private facts about the plaintiff by the defendant. The first branch is not applicable because the fact that the defendant is using the plaintiff's image for profit (such as television program ratings) is not sufficient; liability is generally limited to the use of plaintiff's picture or name in connection with the promotion or advertisement of a product or service. The second branch is not applicable because the intrusion must be into something private; taking video of someone on a public street is not actionable. The third branch is not applicable because nothing suggests that the video placed the plaintiff in a false light. The fourth branch is not applicable because, again, the video was taken on a public street, so the mother's weeping is not likely to be considered a private fact. Furthermore, the First Amendment likely prohibits recovery for this branch of invasion of privacy when the published matter is newsworthy unless the plaintiff establishes that the defendant acted with actual malice (knowledge of falsity or reckless disregard for the truth), which is not the case here. (A) is incorrect. It is true that the right of privacy is a personal right and does not survive the death of the plaintiff. Here, however, the mother is suing for invasion of her own privacy as well as her son's. The rationale in choice (B) negates any claim for invasion of privacy and is therefore a better choice than (A). (C) is incorrect. Because the video was shot on a public street, there was no requirement that the mother give permission for the video to be used. (D) is incorrect. Even though the mother suffered severe emotional distress, she does not have a viable claim for invasion of privacy, as discussed above.
A shop owner domiciled in State A sued a distributor in a federal district court in State A for breach of a contract. The shop owner sought $100,000 in damages for allegedly defective goods that the distributor had provided under the contract. The distributor is incorporated in State B, with its principal place of business in State C. The distributor brought in as a third-party defendant the wholesaler that had provided the goods to the distributor, alleging that the wholesaler had a duty to indemnify the distributor for any damages recovered by the shop owner. The wholesaler is incorporated in State B, with its principal place of business in State A. The wholesaler has asserted a $60,000 counterclaim against the distributor for payment for the goods at issue, and the distributor has moved to dismiss the counterclaim for lack of subject-matter jurisdiction. Should the motion to dismiss be granted? Press Enter or Space to submit the answerANo, because the wholesaler's and the distributor's principal places of business are diverse.BNo, because there is supplemental jurisdiction over the wholesaler's counterclaim.CYes, because there is no diversity of citizenship between the distributor and the wholesaler.DYes, because there is no diversity of citizenship between the shop owner and the wholesaler.
(B) is correct. There is supplemental jurisdiction because the claim for nonpayment for the goods in issue arises out of a common nucleus of operative fact as the plaintiff's (the shop owner's) claims, and the restrictions contained in the supplemental jurisdiction statute do not apply to these facts. This question tests on the restrictions placed on the use of supplemental jurisdiction when the case is in federal court under diversity jurisdiction and the use of supplemental jurisdiction is inconsistent with the requirements for diversity. The rule to remember here is this: The restrictions on the use of supplemental jurisdiction apply to plaintiffs only. Here, the defendant (the distributor) is the party using supplemental jurisdiction, so the restrictions do not apply. As a result, it does not matter that the defendant and third-party defendant are not diverse, and it does not matter that the amount in controversy requirement is not satisfied. (A) is an incorrect statement of the law. A corporation is a citizen of each state in which it is incorporated (only one in real life) and the one state in which it has its principal place of business. Since the wholesaler and the distributor are both incorporated in State B, complete diversity of citizenship does not exist. (C) is incorrect. Although it is a true statement of law, the answer ignores the potential for supplemental jurisdiction. (D) is incorrect. The shop owner is not asserting any claims against the wholesaler, so the lack of diversity between them is irrelevant.
A homeowner was injured when an automatic cutoff switch failed to function on a snowblower he was using. The cutoff switch had functioned well for a year after he purchased the snowblower but failed after the machine had been improperly repaired by a mechanic. The snowblower's operating manual contained a clear and prominent warning against making the very alteration to the switch mechanism that was made by the mechanic. The mechanic, however, did not have a manual available when he repaired the snowblower. Does the homeowner have a viable claim against the manufacturer of the snowblower for damages? Press Enter or Space to submit the answerANo, because the homeowner was contributorily negligent in failing to furnish the snowblower's manual to the mechanic.BNo, because the injury resulted from a substantial alteration of the snowblower by a third party.CYes, because a defect in the snowblower caused the homeowner's injury.DYes, because the manufacturer should have made the manual available to repair personnel.
(B) is correct. To hold a commercial supplier liable for a product defect, the product must be expected to, and must in fact, reach the user or consumer without substantial change in the condition in which it is supplied. Here, the cutoff switch that failed had functioned well for a year after the homeowner purchased the snowblower but failed after the machine had been improperly repaired by a mechanic. The mechanic's alteration to the switch mechanism, which the manual warned against making, will preclude the homeowner from recovering against the manufacturer. (A) is incorrect. Even if the homeowner were arguably somewhat negligent in failing to furnish the manual to the mechanic, such negligence would be minimal and would not preclude the homeowner from having a claim against the manufacturer. (C) is incorrect. To prove actual cause in a products liability action, the plaintiff must trace the harm suffered to a defect in the product that existed when the product left the defendant's control. The facts do not establish that a defect existed in the snowblower before the mechanic repaired it. (D) is incorrect. The facts establish only that the mechanic did not have a manual available, not that the manufacturer did not make one available. The mechanic's failure to obtain a manual (which he could have borrowed from the homeowner) does not establish any fault on the part of the manufacturer.
Seven years ago, a man conveyed vacant land by warranty deed to a woman, a bona fide purchaser for value. The woman did not record the warranty deed and did not enter into possession of the land. Five years ago, the man conveyed the same land to a neighbor, also a bona fide purchaser for value, by a quitclaim deed. The neighbor immediately recorded the quitclaim deed and went into possession of the land. Two years ago, the neighbor conveyed the land to a friend, who had notice of the prior conveyance from the man to the woman. The friend never recorded the deed but went into immediate possession of the land. The jurisdiction has a notice recording statute and a grantor-grantee index system. If the woman sues to eject the friend, will the woman be likely to succeed? Press Enter or Space to submit the answerANo, because the friend took possession of the land before the woman did.BNo, because the neighbor's title was superior to the woman's title.CYes, because the friend had notice of the conveyance from the man to the woman.DYes, because the woman, unlike the friend, took title under a warranty deed.
(B) is correct. Under a notice recording system, a subsequent bona fide purchaser ("BFP") prevails over a prior grantee who failed to record. A BFP is a purchaser who gives valuable consideration and has no notice of the prior grant. Notice includes actual, record, or inquiry notice. Also, under the shelter rule, a person who takes from a BFP will prevail against any interest that the transferor-BFP would have prevailed against. This is true even where the transferee had actual knowledge of the prior unrecorded interest. Here, the neighbor was a BFP. The woman had not recorded her deed and had not taken possession of the land; the neighbor gave value without any notice of the woman's claim. The neighbor's title was superior to the woman's title because of the recording statute. When the neighbor sold the land to the friend, the friend was protected under the shelter rule despite having actual knowledge of the woman's interest. Thus, the friend's title is also superior to that of the woman. (A) is incorrect. Possession isn't required to establish title. The woman did not need to take possession before the friend to prevail; she needed to do something to put the first purchaser (the neighbor) on notice. If the woman had recorded her deed, the neighbor and the friend would not have had a valid claim to the property even if the woman had never taken possession of the property. Record notice would have put the neighbor on notice of her title, and he could not have been a BFP. The friend, taking from the neighbor, would not have been covered by the shelter rule. (If she did not record but took possession, that also would have put the neighbor on inquiry notice and he would not be a BFP.) (C) is incorrect. Under the shelter rule, a person who takes from a BFP will prevail against any interest that the transferor-BFP would have prevailed against. This is true even where the transferee had actual knowledge of the prior unrecorded interest. Here, the neighbor was a BFP and the friend took from the neighbor. Thus, even though the friend had actual knowledge of the woman's deed, he is still protected by the shelter rule and has superior title to the property. (D) is incorrect. The type of deed does not affect who has title. A quitclaim deed has the same effect as a warranty deed in terms of conveying title. The differences among the types of deeds have to do with remedies available against the grantor if title turns out to be defective. A warranty deed gives the grantee contractual promises with respect to title, and the quitclaim deed gives no promises.
A construction worker was working at the construction site of a new building. An open elevator, which had been installed in the building by the elevator manufacturer, was used to haul workers and building materials between floors. While the worker was riding the elevator, it stalled between floors due to a manufacturing defect in the elevator. The worker called for assistance and was in no danger, but after waiting 15 minutes for help, he became anxious and jumped 12 feet to get out. He severely injured his back when he landed. In an action by the worker against the elevator manufacturer to recover for his back injury, is the worker likely to obtain a judgment for 100% of his damages? Press Enter or Space to submit the answerANo, because such risks are inherent in construction work.BNo, because the worker was not in danger while on the stalled elevator.CYes, because the elevator stalled due to a manufacturing defect.DYes, because the worker was falsely imprisoned in the stalled elevator.
(B) is correct. Under contributory negligence principles, a plaintiff is required to act as a reasonably prudent person, the same as the defendant. A plaintiff who fails to act reasonably and who negligently contributes to his injury will have his recovery reduced in jurisdictions following a pure comparative negligence rule (which is the rule applicable to MBE questions). In a case where contributory negligence is shown, the trier of fact weighs the plaintiff's negligence against that of the defendant and reduces the plaintiff's damages accordingly. Here, the worker was in no danger in the stalled elevator but nevertheless jumped 12 feet to the ground after waiting 15 minutes. The trier of fact is likely to find that the worker was at least partly at fault for his injury by jumping from that height. Hence, he will not obtain a judgment for 100% of his damages. (A) is incorrect. The defense of assumption of risk requires that the plaintiff must have known of the risk and voluntarily assumed it. Here, a manufacturing defect in a construction elevator is not likely an inherent risk in construction work, and the worker will not be deemed to have assumed such a risk. (C) is incorrect. While the fact that the elevator stalled due to a manufacturing defect may allow the worker to bring a products liability action based on strict liability, most comparative negligence states apply their comparative negligence rules to strict products liability actions. Hence, the worker's unreasonable conduct in jumping off the elevator will likely prevent him from recovering 100% of his damages, even if the worker brings an action based on strict liability. (D) is incorrect. False imprisonment is an intentional tort, and there is no evidence that the elevator manufacturer intended to confine the plaintiff.
In an effort to counteract a steep increase in juvenile crime, a state enacted a law terminating the parental rights of any state resident whose child under 16 years of age is convicted of a violent crime in the state. The law directs the state juvenile court to enter a termination order in such a case after the parent has been afforded notice and an opportunity for a hearing at which the only relevant issues are the age of the child and whether the child has been convicted of a violent crime in the state. Is the state law constitutional? Press Enter or Space to submit the answerANo, because the law is not narrowly tailored to serve a substantial state interest.BNo, because the law is not necessary to serve a compelling state interest.CYes, because a state's police power authorizes it to punish criminal behavior with appropriate sanctions.DYes, because the law is rationally related to a legitimate state interest.
(B) is correct. Under the Due Process Clause, if government action impacts a fundamental right, the action is valid only if it passes strict scrutiny - that is, the government must prove that the action is necessary to achieve a compelling state interest. This is a very difficult hurdle to clear. The U.S. Supreme Court has held that the Constitution includes a fundamental right of privacy, and the fundamental right of privacy includes the right of parents to the companionship, care, custody, and control of their children. This fundamental right surely extends to instances where, as here, the state seeks to terminate companionship/parental rights. Thus, the only choice that reflects the strict scrutiny standard is (B). While whether the standard was met is a factual determination, as indicated above, the standard is very hard to meet. Surely, there are ways to curtail juvenile crime other than automatically terminating parental rights on proof that the parent's child committed a violent crime. For example, perhaps parenting classes would be equally or more effective. (A) is incorrect. Although it comes to the correct result (that is, the law is unconstitutional), it sets forth an intermediate scrutiny test, which is not used in substantive due process analysis. (C) is incorrect. Although it is true that states have the power to punish criminal behavior, their punishments and procedures must not violate any constitutional provision or federal law. As discussed above, the automatic termination of parental rights here violates the Due Process Clause. (D) is incorrect. Although it probably is true that the law here is rationally related to reducing juvenile crime (which is a legitimate government interest), as indicated above, the rational basis test does not apply when a fundamental right is impacted by the government action.
A brick mason was hired by a builder under a written one-year contract, at an annual salary of $45,000, with employment to begin on March 1. Because the builder was unable to secure enough building contracts to keep all its employees busy during the season beginning March 1, it notified the brick mason on February 15 that it could not afford to employ him as a mason. At the same time, however, the builder offered to employ the mason, for the same contract period, as a night guard at an annual salary of $25,000. The mason declined the offer and remained unemployed during the year. No employment for brick masons was available in the community during the year, but the mason could have obtained other employment as a day laborer that would have paid up to $25,000 a year. At the end of the year, in an action against the builder for breach of contract, how much, if anything, is the mason entitled to recover? Press Enter or Space to submit the answerA$20,000 (the $45,000 contract price less the $25,000 the mason could have earned in other employment).B$45,000 (the contract price).CNothing, because the builder did not act in bad faith when it discharged the mason.DNothing, because the mason did not mitigate his damages.
(B) is correct. When an employer breaches an employment contract, the standard measure of the employee's damages is the full contract price. However, the nonbreaching party cannot recover damages that could have been avoided with reasonable effort. If the breaching employer can prove that a comparable job in the same locale was available, then contract damages against that breaching employer for lost wages will be reduced by the wages that the plaintiff would have received from that comparable job. Here, there was no comparable employment available in the community for the mason. The only jobs available, a night guard or a day laborer, were not the same type of work as a brick mason and were at a significantly lower salary. (A) is incorrect. The mason was not obligated to accept employment in a different field at a much lower salary to mitigate his damages. (C) is incorrect. The builder is liable for damages because it breached the employment contract with the mason. The builder's reasons for breaching, even if in good faith, do not excuse the breach. (D) is incorrect. No comparable employment was available in the community during the year and, as stated above, the mason was not obligated to accept employment in a different field at a much lower salary to mitigate his damages.
A mother took her five-year-old child to a hospital emergency room for treatment. A doctor on the hospital staff molested the child while treating her. At the time, no one was in the treatment room except the doctor and the child; the mother had left the room to get a cup of coffee. Two weeks later, when the child told her mother what had occurred in the treatment room, the mother suffered severe emotional distress that caused her to become physically ill. In an action against the doctor by the mother on her own behalf to recover for intentional infliction of emotional distress, is the mother likely to prevail? Press Enter or Space to submit the answerANo, because the mother was contributorily negligent in leaving the child alone with the doctor during treatment.BNo, because the mother was neither the direct victim of the doctor's conduct nor a contemporaneous witness.CYes, because the doctor's conduct was outrageous.DYes, because the mother's distress was the natural and foreseeable consequence of the doctor's conduct.
(B) is correct. When the defendant intentionally causes severe, physical harm to a third person and the plaintiff suffers severe emotional distress because of her relationship to the injured person, the elements of intent and causation in an action for intentional infliction of emotional distress may be harder to prove. To establish these elements in such cases, the plaintiff is generally required to show the following: (1) the plaintiff was present when the injury occurred to the other person; (2) the plaintiff was a close relative of the injured person; and (3) the defendant knew that the plaintiff was present and a close relative of the injured person. Here, the mother was not present when the doctor molested her child, so she cannot recover under these criteria. Note that the plaintiff does not need to establish presence or a family relationship if she shows that the defendant had a design or purpose to cause severe distress to the plaintiff, but there is no evidence of that in the facts. (A) is incorrect. Contributory negligence is not a defense to intentional torts such as intentional infliction of emotional distress. (C) is incorrect. While the doctor's conduct was outrageous and the mother suffered severe emotional distress, the mother has not established the elements of intentional infliction of emotional distress, as discussed above. (D) is incorrect. In cases where intentional physical harm is inflicted on a third person, the plaintiff must establish the criteria discussed above to establish intent and causation for her recovery for intentional infliction of emotional distress, which the mother was not able to do here.
A defendant is on trial in federal court for bank robbery. Before the police had any suspects, a police officer interviewed an eyewitness at the police station and showed her a "mug book" containing dozens of photographs. The eyewitness identified the defendant's photograph as that of the robber. At trial, the eyewitness surprises the prosecutor by testifying that she is unable to identify the defendant as the robber. The prosecutor calls the officer to testify that the eyewitness identified the defendant from the photograph in the police station. The eyewitness remains present in the courthouse and can be recalled. Is the officer's testimony admissible? Press Enter or Space to submit the answerANo, because the eyewitness was unable to identify the defendant at trial.BNo, because the eyewitness's testimony has disappointed the prosecutor but has not affirmatively harmed the prosecution's case.CYes, because the eyewitness's statement of identification as reported by the officer is not excluded by the hearsay rule.DYes, because the hearsay rule does not exclude out-of-court statements if a declarant testifies and is available for cross-examination.
(C) is correct. A declarant's prior statement identifying someone she perceived earlier is not hearsay under the Federal Rules of Evidence if the declarant is now testifying at trial and subject to cross-examination about the prior statement. Here, the eyewitness testified at trial and can be recalled and cross-examined, so her prior statement identifying the defendant as the robber is admissible. (A) is incorrect. Even though the eyewitness was unable to identify the defendant at trial, her prior statement of identification is admissible because she is testifying and subject to cross-examination about the statement. (B) is incorrect. A testifying witness's prior statement of identification is admissible regardless of whether the witness's current testimony has affirmatively harmed the examiner's case. (D) is incorrect. This answer choice is too broad. Only certain out-of-court statements of a testifying witness are categorized as nonhearsay-prior inconsistent statements made under oath, prior statements of identification, and prior consistent statements that rehabilitate the witness. Otherwise, the general rule is that a declarant-witness's out-of-court statements are hearsay if offered for their truth and must fall within an exception to be admissible.
A fumigation company was hired to eliminate pests in one of two buildings in a condominium complex that shared a common wall. The owners of the complex told the fumigation company that the common wall separating the infested building from the uninfested building was an impenetrable fire wall. The fumigation company did its own thorough inspection and determined that the buildings were indeed completely separated by the wall. Residents of the condominium units in the building that was to be sprayed were told to evacuate, but the residents of the uninfested building were told that they could remain while the other building was treated. During and shortly after the fumigation, in which a highly toxic chemical was used, many residents of the uninfested building became sick. It was determined that their illnesses were caused by the fumigation chemical. In fact, there was a hole in the fire wall separating the two buildings, but because it could only be observed from a specific position in the crawl space underneath the floor of the uninfested building, it had not been discovered by either the fumigation company or any previous building inspector. Are the residents of the uninfested building likely to prevail in a tort action against the fumigation company? Press Enter or Space to submit the answerANo, because the condominium complex owners were responsible for accurately conveying the condition of their buildings.BNo, because the fumigation company exercised a high level of care.CYes, because the fumigation company can be held strictly liable for its activity.DYes, because the fumigation company put a dangerous product into the stream of commerce.
(C) is correct. A defendant may be held strictly liable for an activity that is deemed abnormally dangerous. The courts generally impose two requirements for finding an activity to be abnormally dangerous: (i) the activity must create a foreseeable risk of serious harm even when reasonable care is exercised by all actors; and (ii) the activity is not a matter of common usage in the community. Fumigating is commonly held to be an abnormally dangerous activity for which strict liability is imposed. (A) is incorrect. The fumigation company did its own thorough inspection of the buildings and did not rely on the information from the condominium complex owners. Even had that been the case, the fumigation company would be strictly liable for the harm that occurred. (B) is incorrect. Because the fumigation company is strictly liable for its activity, it does not matter that it exercised a high level of care. (D) is incorrect. The fumigation company used a dangerous product but it did not put it into the stream of commerce. Hence, its strict liability is based on engaging in an abnormally dangerous activity rather than supplying a dangerous and defective product.
A patient who had suffered a severe fracture of her leg was treated by an orthopedist, who set the patient's leg and put it in a cast. When the leg continued to bother the patient six months later, she consulted a second orthopedist in the same town. The second orthopedist surgically inserted a pin to facilitate healing. The patient brought a malpractice action against the first orthopedist, claiming that he should have surgically inserted a pin at the time of initial treatment. The only evidence that the patient offered in support of her malpractice claim was the testimony of the second orthopedist, as follows: In response to the question "Would you have inserted a pin initially?" the second orthopedist testified, "I personally would not have been satisfied that the leg would heal properly without a pin." At the close of the patient's evidence, the first orthopedist moved for judgment as a matter of law. Should the motion be granted? Press Enter or Space to submit the answerANo, because the patient has introduced evidence that the first orthopedist failed to give the care that the second orthopedist would have provided.BNo, because the second orthopedist practices in the same town and field of specialty as the first orthopedist.CYes, because the patient has failed to introduce evidence that the first orthopedist's care fell below the professional standard of care.DYes, because the second orthopedist also treated the patient and is thus not sufficiently objective to offer expert testimony.
(C) is correct. A person who is a professional (e.g., doctor) is required to possess and exercise the knowledge and skill of a member of the profession or occupation in good standing. For doctors, most courts will apply a national standard of care to evaluate their conduct. This standard of care generally is established by expert testimony. Here, the only evidence the patient offered in support of her malpractice claim was the testimony of the second orthopedist. However, that orthopedist stated only that he personally would have inserted a pin to make sure the leg healed properly. He did not testify as to what treatment conforms to the requisite standard of care or whether the first orthopedist had failed to conform to that standard of care by not inserting a pin initially. Hence, the patient has not presented sufficient evidence of breach of duty to withstand a motion for judgment as a matter of law. (A) is incorrect. Evidence that the second orthopedist would have provided different care is not enough to establish breach of duty, as discussed above. (B) is incorrect. As discussed above, most courts apply a national standard of care to evaluate the standard of care of doctors, particularly if they are specialists. Thus, the fact that the second orthopedist practices in the same town as the first orthopedist is irrelevant. (D) is incorrect. The fact that the second orthopedist treated the patient does not preclude him from testifying to the appropriate standard of care.
A state owned and operated an electric power system, which included a nuclear power plant. In order to ensure the availability of sites for the disposal of spent fuel from the nuclear power plant, the state refused to supply electric power to out-of-state purchasers residing in states that would not accept spent fuel from the plant for storage or disposal. Assume that no federal statute applies. Which of the following is the strongest argument that the state's action is constitutional? Press Enter or Space to submit the answerAA state may condition the sale to out-of-state purchasers of any products produced in that state on the willingness of those purchasers to bear the fair share of the environmental costs of producing those products.BThe generation of electricity is intrastate by nature and therefore subject to plenary state control.CThe state itself owns and operates the power system, and therefore its refusal to supply power to out-of-state purchasers is not subject to the negative implications of the Commerce Clause.DThe state's action is rationally related to the health, safety, and welfare of state citizens.
(C) is correct. As a general rule, under the negative implications of the Commerce Clause (sometimes called the Dormant Commerce Clause), a state may not adopt a law or take an action that discriminates against interstate commerce. While a decision to not sell power to certain out-of-state customers appears to run afoul of this rule, in this case, the decision falls squarely within an exception to the prohibition - a state may prefer its own citizens or certain states when the state is acting as a market participant (e.g., buying or selling products). Thus, the strongest argument for finding the state's action constitutional is that the state is operating the power system. (A) is incorrect because it is far too broad to be true. A state may not condition sales of all products produced in the state on the willingness of purchasers to share environmental costs - a state may not place downstream regulations on products when they reach other states. (B) also is untrue. While electricity might be produced within a state, production and sale of electricity is a commercial activity subject to the federal commerce power; a state's regulation of commercial activity is not plenary. (D) is incorrect. A state cannot justify an apparent breach of the negative implications of the Commerce Clause merely by showing that the breach is rationally related to a police power concern (that is, health, safety, or welfare of citizens); the action must be necessary to achieve a compelling interest or some exception (such as the market participant exception) must apply.
A plaintiff sued the manufacturer of a slicing machine for negligent design, after the machine cut off the plaintiff's finger. The plaintiff claims that while he was cleaning the machine, two wires came into contact with each other and caused the machine to turn on. At trial, the manufacturer has offered evidence that it was unreasonably expensive to design the machine so that the wires could not come into contact. In rebuttal, the plaintiff offers evidence that after this action was filed, the manufacturer redesigned the machine to prevent the wires from coming into contact. Is evidence of this change in design admissible? Press Enter or Space to submit the answerANo, because the change in design may have been unrelated to this type of accident.BNo, under the rule regarding remedial measures that encourages manufacturers to make their products safer.CYes, as evidence tending to show that the machine could be designed to keep the wires from coming into contact.DYes, as evidence tending to show that the manufacturer was negligent because its initial design failed to prevent the wires from coming into contact.
(C) is correct. As a matter of public policy, evidence of repairs or other precautionary measures made following an injury is inadmissible to prove negligence, culpable conduct, a defect in a product or its design, or a need for a warning or instruction. The purpose of the rule is to encourage people to make such repairs. However, such evidence may be admissible for other purposes. One permissible purpose is to rebut a claim that the repair or precaution was not feasible. Here, the manufacturer has introduced evidence that it was unreasonably expensive to design the machine so that the wires would not come into contact-but they did in fact implement this new design. Thus, the plaintiff may introduce evidence of the new design to rebut the manufacturer's claim that it was not feasible. (A) is incorrect. It is immaterial whether the change in design came as a result of the plaintiff's accident; it is still relevant to rebut the manufacturer's evidence that the change was not feasible. (B) is incorrect because it is too broad. The policy exclusion for subsequent remedial measures prohibits such evidence when offered to prove negligence, culpable conduct, a defect in a product or its design, or a need for a warning or instruction. However, such evidence is still admissible to rebut a contention that the remedial measure was not feasible. (D) is incorrect because evidence of subsequent remedial measures is inadmissible to prove negligence. However, the evidence is admissible in this case to prove that it was feasible to redesign the machine.
A defendant who is an accountant has been charged with fraud for allegedly helping a client file false income tax returns by shifting substantial medical expenses from one year to another. The defendant has pleaded not guilty, claiming that he made an honest mistake as to the date the expenses were paid. At trial, the prosecutor offers evidence of the defendant's involvement in an earlier scheme to help a different client falsify tax returns in the same way. Is the evidence of the defendant's involvement in the earlier scheme admissible? Press Enter or Space to submit the answerANo, because it is impermissible character evidence.BNo, because it is not relevant to the issues in this case.CYes, to show absence of mistake.DYes, to show the defendant's propensity to commit the crime.
(C) is correct. Because the defendant is claiming that he made an honest mistake, his prior involvement in a similar scheme is relevant to show that there was no mistake. (A) is incorrect. Although character evidence is usually not admissible to show a party's propensity to act in accordance with the trait, evidence of a party's specific acts may be admitted if independently relevant (e.g., to show motive, intent, identity, or lack of mistake). Here the evidence is admissible to show a lack of mistake. (B) is incorrect. Evidence is relevant if it has any tendency to make the existence of a fact of consequence to the determination of the action more probable or less probable than it would be without the evidence. This is a low bar, and here the evidence makes it less likely that the defendant made a mistake during the events at issue in this case. Thus, it is relevant. (D) is incorrect. The prosecution generally may not introduce evidence of a defendant's character to show his propensity to commit the charged crime. Thus, if the prosecution were offering this evidence to show that the defendant is dishonest and prone to committing fraud, it would be inadmissible. However, evidence of a party's specific acts may be admitted for an independent purpose, and here it is relevant to counter the defendant's assertion that he made a mistake.
A security guard, dressed in plain clothes, was working for a discount store when a customer got into a heated argument with a cashier over the store's refund policy. Without identifying himself as a security guard, the security guard suddenly grabbed the customer's arm. The customer attempted to push the security guard away, and the security guard knocked the customer to the floor, causing injuries. The customer sued the discount store for battery on a theory of vicarious liability for the injuries caused by the security guard. The store filed an answer to the customer's complaint, asserting the affirmative defense of contributory negligence. The customer has moved to strike the affirmative defense. Traditional rules of contributory negligence apply. Should the trial court grant the customer's motion? Press Enter or Space to submit the answerANo, because contributory negligence is an affirmative defense to a cause of action based on vicarious liability.BNo, because the customer should have known that his argument with the cashier might provoke an action by a security guard.CYes, because contributory negligence is not a defense to battery.DYes, because the customer did not know that he was pushing away someone who was employed as a security guard.
(C) is correct. Contributory negligence is a defense to negligence actions but not to intentional torts actions. Battery is an intentional tort for which contributory negligence is not a defense. Hence, the court should grant the customer's motion to strike the affirmative defense. (A) is incorrect. A cause of action based on vicarious liability may arise not just from an employee's negligence but also from an employee's intentional conduct in certain circumstances. Here, the vicarious liability allegation is based on the security guard's intentional tort of battery, for which contributory negligence is not a defense. (B) is incorrect. Even if the customer should have known that a security guard would intervene and was therefore contributorily negligent, that would not be a defense to battery. (D) is incorrect. The fact that the customer did not know that he was pushing a security guard does not address the store's defense, which is that the customer should have known that the person who intervened could have been a security guard and therefore was contributorily negligent. Choice (C) addresses why the defense will fail.
A man borrowed money from a lender and mortgaged land that he owned to secure repayment of the loan. Before he had completely repaid the loan, the man conveyed the land to an investor, who expressly assumed the loan. The note and mortgage did not contain a due-on-sale clause. After the investor had made several payments on the loan, she defaulted on two payments. The lender notified the man and the investor of its intention to accelerate the loan pursuant to the terms of the note and mortgage unless the default was cured within 60 days. When neither the man nor the investor made the required payment, the lender accelerated the loan and initiated foreclosure proceedings, naming both the man and the investor as party defendants. The foreclosure sale resulted in a deficiency. The lender has sought a deficiency judgment against only the man, because the investor has become insolvent in the meantime. Will the court likely find the man liable for the deficiency? Press Enter or Space to submit the answerANo, because the investor's express assumption of the loan released the man from liability.BNo, because the lender must first seek to obtain a deficiency judgment against the investor.CYes, because even after the assumption, the man remains liable as a surety of the investor in the absence of a release from the lender.DYes, because the note and mortgage did not contain a due-on-sale clause.
(C) is correct. If a mortgagor sells property subject to a mortgage, the grantee takes subject to the mortgage, which remains on the land. If the grantee signs an agreement promising to pay the mortgage loan ("assuming the loan"), he becomes primarily liable to the lender, while the original mortgagor becomes secondarily liable as a surety. The mortgagee may opt to sue either the grantee or the original mortgagor on the debt. Here the man conveyed the land to the investor who expressly assumed the mortgage. That fact means that the investor is primarily liable, but the man remains secondarily liable. If the mortgagee opts to sue the original mortgagor, that party may in turn sue the assuming mortgagor, who is primarily liable. That won't help the man here because the investor is insolvent. (A) is incorrect. Assumption of a mortgage by a grantee makes the grantee primarily liable for the mortgage but does not release the original mortgagor. The original mortgagor remains secondarily liable as a surety. Thus, the investor's assumption of the loan did not release the man from liability. (B) is incorrect. If the proceeds of a foreclosure sale are insufficient to satisfy the mortgage debt, the mortgagee can bring a personal action for the deficiency against either the grantee who assumed the mortgage or the original mortgagor on the debt. The mortgagee is not required to sue the assuming mortgagor first. (D) is incorrect. A due-on-sale clause allows the lender to demand full payment of the loan if the mortgagor transfers any interest in the property without the lender's consent. The lack of such a clause here is irrelevant to the man's liability for the deficiency.
A mechanic sued his former employer in federal court, claiming that the employer had discharged him because of his age in violation of federal law. The employer answered, denying the claims and promptly moving for summary judgment. In support of the motion, the employer attached the mechanic's employment evaluations for the past three years, which rated his skills and performance as poor and culminated in a recommendation for his discharge. What is the mechanic's best argument to defeat the summary judgment motion? Press Enter or Space to submit the answerAThe allegations in the complaint conflict with the mechanic's employment evaluations, raising a genuine dispute as to material facts.BThe employer cannot rely in his motion on matters outside the pleadings.CThe essential facts are unavailable to the mechanic and therefore discovery is required.DThe motion was filed before the close of discovery.
(C) is correct. If the mechanic (the nonmovant) shows by affidavit or declaration that he cannot present facts essential to justify his opposition to the summary judgment motion, Rule 56(d) authorizes him to ask the court to defer action or deny the motion to allow time to obtain affidavits or declarations or to take discovery. The employer moved for summary judgment right after answering and before any discovery. That timing would support defeating the summary judgment motion at this time. (A) is incorrect. Under Rule 56(c), a party asserting that a fact is genuinely disputed must support the assertion by citing particular parts of the record, including affidavits or declarations, stipulations, or discovery materials. The mechanic cannot simply rely on the complaint allegations to rebut the employer's evidence but must support his factual position with his own evidence that a factual dispute exists. If he cannot do so, Rule 56(d) authorizes him to ask the court to defer action or deny the summary judgment motion to allow time to obtain affidavits or declarations or to take discovery. (B) is incorrect. The function of a summary judgment motion is to allow additional evidence outside the pleadings to show that there is no genuine dispute of fact and that the movant is entitled to judgment as a matter of law. Rule 56(c)(1) enumerates the types of materials that the moving party may use to support a summary judgment motion, including documents. If the mechanic (the nonmovant) shows by affidavit or declaration that he cannot present facts essential to justify his opposition to the summary judgment motion, Rule 56(d) authorizes him to ask the court to defer action or deny the motion to allow time to obtain affidavits or declarations or to take discovery. (D) is incorrect. The fact that a summary judgment motion is filed before the close of discovery does not require the court to deny it. Under Rule 56(b), a party may file the motion at any time until 30 days after the close of all discovery. The problem here is that the employer filed the motion before discovery commenced, thus providing the mechanic an argument to defer action or deny the motion under Rule 56(d).
A landlord leased a building to a tenant for a 10-year term. Two years after the term began, the tenant subleased the building to a sublessee for a 5-year term. Under the terms of the sublease, the sublessee agreed to make monthly rent payments to the tenant. Although the sublessee made timely rent payments to the tenant, the tenant did not forward four of those payments to the landlord. The tenant has left the jurisdiction and cannot be found. The landlord has sued the sublessee for the unpaid rent. There is no applicable statute. If the court rules that the sublessee is not liable to the landlord for the unpaid rent, what will be the most likely reason? Press Enter or Space to submit the answerAA sublessee is responsible to the landlord only as a surety for unpaid rent owed by the tenant.BThe sublease constitutes a novation of the original lease.CThe sublessee is not in privity of estate or contract with the landlord.DThe sublessee's rent payments to the tenant fully discharged the sublessee's obligation to pay rent to the landlord.
(C) is correct. In a sublease, the tenant transfers a right of possession for a time shorter than the balance of the leasehold. Therefore, the sublessee and the tenant are in privity of estate with each other, but only the tenant remains in privity of estate with the landlord. There also is no privity of contract between the sublessee and the landlord, because the sublessee made no promise, either to the landlord or to the tenant, to pay rent to the landlord. Lacking privity, the sublessee is not liable to the landlord for the rent. Although privity may not be required under an equitable servitude theory, a finding for the sublessee would mean that the court did not use such a theory. (A) is incorrect. As stated above, there is no privity of contract between the sublessee and the landlord, because the sublessee made no promise, either to the landlord or to the tenant, to pay rent to the landlord. Lacking privity, the sublessee is not liable to the landlord for the rent and also is not a surety for the tenant. (B) is incorrect. A novation occurs when a tenant seeks to avoid future liability for rent after an assignment and the landlord agrees to release the tenant from such liability. An assignment occurs when the tenant transfers the entire period of time remaining on the lease agreement. Here, the tenant only transferred a portion of the remaining time on the lease agreement, and the tenant did not seek a release or novation from the landlord. (D) is incorrect. The sublessee had no obligation to pay rent to the landlord. In a sublease, the tenant transfers a right of possession for a time shorter than the balance of the leasehold. Therefore, the sublessee and the tenant are in privity of estate with each other, but only the tenant remains in privity of estate with the landlord. There also is no privity of contract between the sublessee and the landlord, because the sublessee made no promise, either to the landlord or to the tenant, to pay rent to the landlord. Lacking privity, a sublessee is not liable to the landlord for the rent.
A state initiated a criminal prosecution against the owner of a video store for selling a video that consisted entirely of pictures of nude sunbathers on a beach in a foreign country where nude public sunbathing is common. The state charged that selling the video violated its anti-obscenity law. The store owner defended on the ground that the prosecution violated his constitutional right to freedom of speech. Should the store owner prevail in this defense? Press Enter or Space to submit the answerANo, because the store owner is engaged in the commercial sale of the video, which is not protected by the First and Fourteenth Amendments.BNo, because the video consists entirely of portrayals of nudity, appeals to the prurient interest of viewers, and lacks serious social value as a whole.CYes, because mere portrayals of nudity are insufficient to justify a finding that the video is obscene as a matter of constitutional law.DYes, because the portrayals of nudity occurred outside the United States, and therefore the state lacks a compelling interest in applying its anti-obscenity law to the sale of the video.
(C) is correct. The Supreme Court has defined obscenity as a description or depiction of sexual conduct that, taken as a whole by the average person, applying contemporary community standards: (1) appeals to the prurient interest in sex; (2) portrays sex in a patently offensive way; and (3) using a national standard, does not have serious literary, political, or scientific value. The video here merely has pictures of nude sunbathers, and there is no mention of a depiction of sexual conduct. Therefore, it does not fall within the definition of obscenity and the store owner should prevail. (A) is incorrect because it implies commercial speech is not protected at all by the First Amendment, and that is simply not true. Commercial speech is protected, although a different standard of review applies as compared to other areas of speech. (B) is incorrect. It is tempting because it sounds a lot like the test set out above, but its conclusion is wrong - pictures of nude people sunbathing do not portray sexual conduct and do not exhibit a prurient interest (that is, excessive or unwholesome interest in sex). (D) is a complete red herring - the fact that the portrayals in a movie, pictures, books, etc., may be of events outside the United States would do nothing to prevent the portrayals from being deemed obscene. Regardless of where the events occurred or appear to occur, if the portrayals fall within the above definition, they are obscene, and if they do not fall within the above definition, they are not obscene.
A builder contracted in writing to construct a small greenhouse on a homeowner's property for $20,000, payable upon completion. After the builder had spent $9,000 framing the greenhouse and an additional $1,000 for materials not yet incorporated into the greenhouse, the homeowner wrongfully ordered the builder to stop work. The builder then resold the unused materials that he had already purchased for the greenhouse to another contractor for $1,000. At the time the homeowner stopped the work, it would have cost the builder an additional $5,000 to complete the project. The partially built greenhouse increased the value of the homeowner's property by $3,000. In a suit by the builder against the homeowner, how much is the builder likely to recover? Press Enter or Space to submit the answerA$3,000, the increase in the value of the homeowner's property.B$10,000, the total cost expended by the builder at the time of the breach.C$14,000, the total cost expended by the builder ($10,000) plus the builder's expected profit ($5,000), minus the loss avoided by the resale of the unused materials ($1,000).D$15,000, the contract price ($20,000) minus the costs saved by the breach ($5,000).
(C) is correct. The builder is likely to recover $14,000. In a construction contract, if the property owner breaches the contract during construction, the builder is entitled to any profit he would have derived from the contract plus any costs he has incurred to date. If the builder has mitigated his damages, any losses that are avoided must be subtracted from this amount. (A) is incorrect. In a construction contract, when the property owner breaches before the construction is completed, the builder's damages are not measured by the increase in value of the homeowner's property. (B) is incorrect. The builder is also entitled to the profit he would have made if the contract had been performed. (D) is also incorrect. The formula for awarding a builder damages for a breach during a construction contract can also be stated as the contract price minus the cost of completion, which would be $15,000. However, this answer fails to account for the $1,000 of damages the builder avoided by reselling the unused materials.
A police officer was employed on a city's police force for 10 years. When the officer accepted the job, the city's employee benefit plan provided a death benefit to the spouse of any employee who died as a result of any job-related injury. Last year, the city amended its employee benefit plan to deny its death benefit in cases where the death "was caused by the employee's refusal to accept, for any reason other than its excessive risk to life or health, reasonably available medical care prescribed by a physician." After this amendment took effect, the officer was shot while on duty. Because of a sincerely held religious belief, the officer refused to allow a prescribed blood transfusion and, as a result, died from loss of blood. When the officer's spouse applied for the death benefit, the city denied the application on the basis of the amendment to the employee benefit plan. The officer's spouse has challenged the amendment, claiming that, as applied to the officer, it violated the officer's constitutional right to the free exercise of religion. Is the court likely to find the amendment to the employee benefit plan constitutional as applied to the officer? Press Enter or Space to submit the answerANo, because it effectively discriminates against a religious practice.BNo, because it violates the vested contractual rights of city employees who were hired before the amendment took effect.CYes, because it does not single out religious reasons for the denial of benefits and is a reasonable limitation on the award of such benefits.DYes, because it imposes a condition only on the award of a government benefit and does not impose a penalty on an individual's conduct.
(C) is correct. The court is likely to find the amendment to the employee benefit plan constitutional under the Free Exercise Clause. The Free Exercise Clause prohibits government from punishing someone (which includes denying a benefit) on the basis of the person's religious belief. However, the Supreme Court has held that the prohibition applies only to government conduct that targets conduct specifically because it is religiously motivated; a religiously neutral regulation that incidentally burdens religious conduct is valid, and the government need not include an exemption for religiously motivated instances of the regulated conduct. Here, the city amended its employee benefit plan to deny death benefits when death resulted from an employee's refusal to accept medical care. Nothing in the facts indicates the change was adopted in order to target the employee's religious beliefs; rather, the change seems to be a religiously neutral law that happened to burden the police officer's religious belief against accepting blood transfusions. Therefore, the amendment is constitutional. (A) is incorrect. Incidental discrimination against religious practices is not prohibited by the Free Exercise Clause. (B) is incorrect. This choice raises a Contract Clause issue while the officer's spouse challenged the benefit plan amendment under the Free Exercise Clause. Moreover, the Contract Clause merely limits the ability of states to enact laws that retroactively impair contract rights. Here, nothing indicates the city enacted any law or that the employee had a perpetual right to the death benefit the officer originally had. (D) is incorrect. Its reasoning is wrong - the Free Exercise Clause applies to denial of benefits as well as impositions of penalties.
At trial in an action for personal injuries suffered in a traffic accident, the plaintiff first calls the defendant as an adverse party. The plaintiff then calls a witness who was a passenger in the plaintiff's car but who also happens to be the defendant's former employer. On direct examination, the witness testifies to how the accident occurred and also expresses his opinion that the defendant is not a truthful person. Which one of the following areas of questioning is most likely to be held beyond the proper scope of cross-examination? Press Enter or Space to submit the answerAIn letters to prospective employers, the witness has described the defendant as very honest and dependable.BThe defendant recently filed an action against the witness for breach of contract.CThe plaintiff's injuries were not as serious as the plaintiff is claiming.DThe witness has been falsifying his income tax returns.
(C) is correct. The scope of cross-examination is generally limited to: (i) matters brought up on direct examination; and (ii) matters concerning the witness's credibility (i.e., impeachment). Here, the witness's testimony on direct examination concerned how the accident happened and the defendant's character for untruthfulness; there is no indication that the witness testified about the plaintiff's injuries. Because the questioning does not concern the subject matter of the direct examination or the witness's credibility, it is most likely to be held beyond the proper scope of cross-examination. The other answer choices describe proper impeachment of the witness. (A) is incorrect. A common method of impeachment is asking the witness about the witness's prior statements that are inconsistent with the witness's current testimony. Here the witness testified as to his opinion that the defendant is an untruthful person. Thus, it would be proper to cross-examine the witness about his inconsistent prior descriptions of the defendant's good character for honesty. (B) is incorrect. A witness may be impeached with evidence that the witness is biased in favor of or against a party in the case. Here, the fact that the defendant recently filed an action against the witness for breach of contract shows that the witness may have a motive to testify against the defendant in this case. (D) is incorrect. A party may impeach a witness by cross-examining the witness about the witness's specific acts of misconduct involving untruthfulness. Thus, it is proper to ask the witness about whether the witness falsified his tax returns.
A buyer sent a signed letter to a seller that stated: "Ship 100 boxes of nails at $3 per box, the price quoted in your circular." The seller mailed the buyer a signed form acknowledgment that agreed to the buyer's terms and stated on the reverse side: "Disputes regarding quality shall be arbitrated." The buyer did not reply to the seller's acknowledgment, and the seller shipped the nails. When the buyer received the nails, it found their quality to be unsatisfactory and sued the seller for breach of warranty. The seller has asked an attorney whether the parties' contract requires arbitration of the buyer's claim. What is the best advice the attorney can provide? Press Enter or Space to submit the answerAA contract was formed pursuant to conduct when the buyer received the nails, and a court would exclude the arbitration provision from the contract.BA contract was formed when the seller mailed its acknowledgment, and the arbitration term became part of the contract.CA contract was formed when the seller mailed its acknowledgment, and the court must decide whether the arbitration term should be excluded as a material alteration of the contract.DNo contract exists, because the arbitration term in the seller's acknowledgment created a counteroffer that the buyer never accepted.
(C) is correct. This is a contract for the sale of goods (nails) so Article 2 of the UCC applies. Under Article 2, the proposal of additional or different terms by the offeree in a definite and timely acceptance does not constitute a rejection and counteroffer, but rather is effective as an acceptance, unless the acceptance is expressly made conditional on assent to the additional or different terms. If both parties to the contract are merchants, the additional terms in the acceptance will be included in the contract unless they materially alter the original terms of the offer; the offer expressly limits acceptance to the terms of the offer; or the offeror has already objected to those terms, or objects within a reasonable time after notice of the terms is received. Whether an alteration is material is a fact question. Here, the seller's acknowledgment contained an additional term - but was not made conditional on its acceptance. Therefore, a contract was formed. The seller is clearly a merchant, and given the size of the order, it appears the buyer is a merchant too. Thus, the court must decide whether the arbitration term should be excluded as a material alteration of the contract. (A) is incorrect. The contract was formed when the seller mailed the buyer the signed acknowledgment form. (B) is incorrect. As stated above, the additional term is not automatically part of the contract. The court must determine if the additional term is a material alteration of the offer. If so, it would not be included in the terms of the contract. (D) is incorrect. Under the common law mirror image rule, a response to an offer that varies the terms of the offer is considered a rejection and a counteroffer. The UCC has abandoned this rule.
A plaintiff sued the manufacturer of his lawn mower, alleging that, as the result of a design defect, he was injured when the mower's blade flew off after striking a sprinkler head in the lawn. At trial, the manufacturer has called as an expert witness a product safety engineer, who testifies that the manufacturer retained him for a fee to test identical mowers and, if his opinion was helpful, to testify in the case. The expert then testifies that he did test the mowers, and that the blade, as designed and installed by the manufacturer, could not fly off in the manner claimed by the plaintiff. Assume that the expert has used a reliable method for reaching his conclusion. Should the court admit the expert's testimony? Press Enter or Space to submit the answerANo, because it goes to an ultimate issue that only the jury can decide.BNo, because the manufacturer paid the expert to render a certain opinion, in violation of rules barring paid testimony.CYes, because expert testimony on such issues of causation is relevant and helpful to the jury.DYes, provided that the plaintiff had notice and an opportunity to participate in the testing process.
(C) is correct. This is the best answer choice because the other answers are clearly wrong. Expert testimony is admissible if: (i) the subject matter is one where scientific, technical, or other specialized knowledge would help the trier of fact understand the evidence or determine a fact in issue; (ii) the opinion is based on sufficient facts or data; (iii) the opinion is the product of reliable principles and methods; and (iv) the expert has reliably applied the principles and methods to the facts of the case. Here, the expert's testimony as to his findings is relevant and helpful to the jury in determining whether the blade flew off in the manner claimed by the plaintiff, and the expert has used a reliable method in reaching his conclusion. (A) is incorrect because an expert is generally not prohibited from giving an opinion as to the ultimate issue in the case (although there is a limited exception for testimony as to a criminal defendant's mental state). (B) is incorrect. Expert witnesses are routinely paid by parties to consult and/or provide testimony in the case. The other party may cross-examine the expert about the compensation to show that the expert is biased, but the compensation has no bearing on the admissibility of the expert's testimony. (D) is incorrect. There is no requirement that an adverse party have notice or an opportunity to participate in an expert's testing process. The plaintiff could have hired his own expert to conduct testing on identical mowers.
An employer offered to pay a terminated employee $50,000 to release all claims the employee might have against the employer. The employee orally accepted the offer. The employer then prepared an unsigned release agreement and sent it to the employee for him to sign. The employee carefully prepared, signed, and sent to the employer a substitute release agreement that was identical to the original except that it excluded from the release any age discrimination claims. The employer signed the substitute release without reading it. Shortly thereafter, the employee notified the employer that he intended to sue the employer for age discrimination. Is the employer likely to prevail in an action seeking reformation of the release to conform to the parties' oral agreement? Press Enter or Space to submit the answerANo, because the employer acted unreasonably by failing to read the substitute release prior to signing it.BNo, because the parol evidence rule will preclude evidence of the oral agreement.CYes, because the employee's fraudulent behavior induced the employer's unilateral mistake.DYes, because the parties were mutually mistaken regarding the contents of the signed release.
(C) is correct. Typically, reformation is not available for unilateral mistake. However, if, as in this case, the writing is inaccurate because of a misrepresentation by the person who created the writing, the innocent party can seek reformation of the contract to reflect the original expressed intent of the parties. (A) is incorrect because a party's failure to read a contract will not necessarily preclude him from obtaining reformation of a contract based on a misrepresentation. Here, the employer's belief that the written contract reflected the parties' agreed terms was not unreasonable. The employer prepared the contract and sent it to the employee to sign. The employer had no reason to suspect that the contract the employee returned was not that same document, but instead was a carefully prepared, altered copy. (B) is incorrect. The parol evidence rule is inapplicable when a party to a written agreement alleges a misrepresentation entitling him to reformation of the agreement. (D) is incorrect because this was not a case of mutual mistake. The employee purposely created a duplicate of the contract with the age discrimination claim release removed and, thus, the mistake as to the contents was unilateral.
A worker was injured when a machine he was using on the job malfunctioned. The worker brought a federal diversity action against both the machine's manufacturer and the company responsible for the machine's maintenance. At trial, the worker submitted a proposed jury instruction on negligence. The court did not accept the proposed instruction and instead gave a negligence instruction that the worker's attorney believed was less favorable and legally incorrect. The attorney did not object to the negligence instruction before it was given. The jury returned a verdict for the defendants. The worker has moved for a new trial on the ground that the court's negligence instruction was improper. What argument has the best chance of persuading the court to grant the motion? Press Enter or Space to submit the answerAIssues of law can be raised at any time.BThe court's negligence instruction was incorrect and the worker's objection to it was preserved when he submitted his proposed negligence instruction.CThe court's negligence instruction was plain error that affected the worker's substantial rights.DThe need for a formal objection to a judicial ruling in order to preserve an argument has been eliminated in the Federal Rules of Civil Procedure.
(C) is correct. Under Federal Rule of Civil Procedure 51(d)(2), a court may consider a plain error in the jury instructions that has not been preserved by an objection if the error affects a substantial right. This represents the only method to have the jury instruction reviewed. (A) is an incorrect statement of the law. Objection and preservation of the objection at trial is required. (B) is incorrect. Federal Rule of Civil Procedure 51 requires a proper objection on the record to preserve an objection to a jury instruction. Here, there was no objection on the record. (D) is an incorrect factual statement. Rule 51 specifically requires an objection on the record.
A hot-air balloon touring company operated near a golf course. The company's property was separated from the golf course by a fence on which the company had posted signs warning people not to enter the property because of the dangers of balloons landing. A golfer on the golf course hit an errant shot onto the company's property, ignored the warning signs, and jumped over the fence to retrieve her golf ball. At about the same time, one of the company's balloons experienced mechanical problems and had to make an emergency landing to avoid crashing. The balloon, which was out of control when it landed, struck the golfer and injured her. The jurisdiction has decided that hot-air ballooning is an abnormally dangerous activity. In an action by the golfer against the company, does the company have any affirmative defenses? Press Enter or Space to submit the answerANo, because the balloon was out of control when it struck the golfer.BNo, because the company was engaged in an abnormally dangerous activity.CYes, because the balloon landed to avoid crashing.DYes, because the golfer assumed the risk by coming onto the company's property.
(D) is correct. A plaintiff who knowingly and voluntarily assumes the risk of injury caused by the defendant's conduct may be subject to the defense of assumption of risk. In a strict liability action, conduct constituting assumption of risk is an affirmative defense; depending on the jurisdiction, it may be a complete defense or it may reduce the plaintiff's recovery under comparative fault principles. Here, the golfer ignored the signs warning of the dangers of balloons landing and jumped over the fence to retrieve her golf ball. Accordingly, even though the company's activity is deemed abnormally dangerous and subject to strict liability, the company can defend on the basis that the golfer assumed the risk by coming onto the company's property. (A) is incorrect. The dangers of out-of-control balloons was the reason for the company to post warning signs on the fence around its property; it does not preclude the company from raising an affirmative defense. (B) is incorrect. While the company was engaged in an abnormally dangerous activity for which it will be strictly liable, it has an assumption of risk defense based on the golfer's conduct. (C) is incorrect. The fact that the balloon landed to avoid crashing is not a defense. Even if the operator of the balloon exercised reasonable care in making the emergency landing, the company would be liable under strict liability principles, except that the golfer assumed the risk through her conduct in entering the property.
A defendant is on trial for theft of a used car that he took for a test drive and did not return. He was arrested in the car two days later. In his defense, the defendant has testified that he had no intention of keeping the car but got caught up in marital problems and simply delayed returning it. The defendant calls a witness to testify that the defendant told him, during the two days, "I'm going to return this car as soon as I work things out with my wife." Is the witness's testimony admissible? Press Enter or Space to submit the answerANo, because it is a self-serving statement by an accused.BNo, because it is hearsay not within any exception.CYes, as a prior consistent statement of the defendant.DYes, as a statement by the defendant of his then-existing state of mind.
(D) is correct. A statement of the declarant's then-existing state of mind (such as motive, intent, or emotion) is admissible as an exception to the hearsay rule. Here, the defendant's statement that he intended to return the car falls squarely within this exception. (A) is incorrect because there is no rule prohibiting self-serving statements; most statements offered by a party are self-serving in some respect. (B) is incorrect. Although the statement is hearsay because it was made outside of the current proceeding and is being offered for its truth-that the defendant intended to return the car-it is admissible under the present state of mind exception. (C) is incorrect. Generally, a witness's testimony cannot be bolstered until the witness has been impeached. Once the witness's testimony has been impeached, prior consistent statements are admissible in certain circumstances to rehabilitate the witness. If admissible to rehabilitate, prior consistent statements are excluded from the definition of hearsay and are admissible as substantive evidence. Here, however, there is no indication that the defendant's testimony has been impeached, so his statement would not be admissible on the basis that it is a prior consistent statement.
A husband and wife acquired land as common law joint tenants with right of survivorship. One year later, without his wife's knowledge, the husband executed a will devising the land to his best friend. The husband subsequently died. Is the wife now the sole owner of the land? Press Enter or Space to submit the answerANo, because a joint tenant has the unilateral right to end a joint tenancy without the consent of the other joint tenant.BNo, because the wife's interest in the husband's undivided 50% ownership in the land adeemed.CYes, because of the doctrine of after-acquired title, or estoppel by deed.D Yes, because the devise to the friend did not sever the joint tenancy.
(D) is correct. Although as a general rule a joint tenant's interest is freely alienable during his or her lifetime without the consent of the other joint tenant, that interest cannot be devised in a will. In this case, on the death of the husband, the wife's interest in the joint tenancy immediately expanded and she became the sole owner of the land as the surviving joint tenant. (A) is incorrect. As a general rule, a joint tenant's interest is freely alienable during his or her lifetime without the consent of the other joint tenant. However, as stated above, a joint tenant's interest cannot be devised in a will. In this case, on the death of the husband, the wife became the sole owner of the land as the surviving joint tenant. (B) is incorrect. The doctrine of ademption applies only when an individual dies testate and attempts to devise land that the testator no longer owns. That doctrine is not applicable here because the wife became the sole owner of the land as the surviving joint tenant. (C) is incorrect. The doctrine of after-acquired title, or estoppel by deed, applies when an individual attempts to convey title (usually by warranty deed) at a time when the individual does not have title to the land but later acquires title to the land. In this case, the husband had an interest in the land but did not have the power to devise it in a will, so the doctrine does not apply.
A pedestrian was injured when hit by a chair that was thrown from an upper-story hotel window. The pedestrian sued the occupants of all the rooms from which the chair might have been thrown. At trial, the pedestrian has been unable to offer any evidence as to the exact room from which the chair was thrown. The defendants have filed a motion for a directed verdict. Should the court grant the motion? Press Enter or Space to submit the answerANo, because it is unreasonable to expect the pedestrian to prove which of the defendants caused the harm.BNo, because of the doctrine of alternative liability.CYes, because a plaintiff always has the burden to prove that a particular defendant's conduct was the factual cause of the plaintiff's physical harm.DYes, because the pedestrian has failed to offer evidence that the defendants jointly engaged in tortious conduct.
(D) is correct. Based on the facts, the pedestrian's action here is likely based on negligence. In a negligence action, the plaintiff has the burden of proving that the defendant breached a duty owed to the plaintiff and that the breach of duty caused the plaintiff's injury. If multiple defendants are acting in concert or jointly engaging in tortious conduct, all of the defendants can be held jointly and severally liable for the injury. Here, the pedestrian has offered no evidence which of the particular defendants threw the chair out the window and has offered no evidence that the defendants were jointly engaged in tortious conduct. Because the pedestrian has failed to establish a prima facie case for negligence, the court should grant the defendants' motion for a directed verdict. (A) is incorrect. In situations where the fact that a particular injury occurred may tend to establish a breach of the duty owed, the plaintiff may be able to rely on res ipsa loquitur to create an inference of liability. However, the plaintiff generally must establish evidence connecting a particular defendant to the negligent conduct to rely on this doctrine. Unless the defendants were jointly engaged in the tortious conduct (which is not the case here), res ipsa loquitur generally may not be used to establish a prima facie case of negligence against any individual party. (B) is incorrect. Under the doctrine of alternative liability or alternative causes, which applies when two or more persons have been negligent but uncertainty exists as to which one caused the plaintiff's injury, once the plaintiff proves that harm has been caused to him by one of the defendants, the burden of proof shifts to defendants, and each must show that his negligence is not the actual cause of the harm. Here, the pedestrian has not shown that all of the defendants were negligent, so this doctrine is inapplicable. (C) is incorrect. The language of this choice is too broad (indicated by the word "always" in the choice). In alternative liability situations the plaintiff will not have the burden to prove that a particular defendant was the factual cause of the harm. However, the facts here do not present such a situation.
A purchaser filed a federal diversity action against a seller, alleging breach of contract. The seller answered the complaint and included as a separate defense an allegation that the purchaser had brought and lost a similar contract claim against a different seller three years earlier, and that this history represented a pattern of filing frivolous lawsuits. The purchaser believes that the earlier lawsuit was factually completely different from the current one and is therefore irrelevant. What is the purchaser's best response to the seller's answer? Press Enter or Space to submit the answerAFile a reply that includes a denial of the separate defense.BMove for sanctions against the seller for asserting a frivolous defense.CMove to amend the complaint to add allegations about the differences between the lawsuits.DMove to strike the separate defense as irrelevant.
(D) is correct. Before responding to a pleading or, if no responsive pleading is permitted, within 21 days after service of the pleading, a party may move to have stricken any insufficient defense, or any redundant, immaterial, impertinent, or scandalous matter. Since the prior lawsuit is irrelevant to the current lawsuit, the purchaser should move to strike the defense. (A) is incorrect. A denial of the defense would be treating the defense as valid. It would leave the defense pending for trial, and evidence about the prior lawsuit in theory could be introduced. It is far better to have the defense stricken prior to trial, as it saves time at trial and prevents the jury from potentially hearing any facts about the prior lawsuit. (B) is incorrect. There is no indication that the seller knew the defense to be a sham, so it is unlikely that the court would impose any other sanction outside of striking the pleading. Absent any evidence of a Rule 11 violation, the matter may be best handled in a more direct manner by moving to strike the pleading. (C) is incorrect. If the lawsuit is irrelevant to the current proceedings, the purchaser should not add allegations about the differences between the two suits. Again, this strategy would be treating the defense as valid, which should not be the purchaser's strategy
A man who owned a business believed that one of his employees was stealing computer equipment from the business. He decided to break into the employee's house one night, when he knew that the employee and her family would be away, to try to find and retrieve the equipment. The man had brought a picklock to open the employee's back door, but when he tried the door, he found that it was unlocked, so he entered. As the man was looking around the house, he heard sounds outside and became afraid. He left the house but was arrested by police on neighborhood patrol. What is the man's strongest defense to a burglary charge? Press Enter or Space to submit the answerAThe back door to the house was unlocked.BThe burglary was abandoned.CThe house was not occupied at the time of his entry.DThe man did not intend to commit a crime inside the house.
(D) is correct. Burglary is the breaking and entering of the dwelling of another at nighttime with the intent to commit a felony therein. Here, the man intended to retrieve the computer equipment that he thought the employee was stealing from his business. Since the man only wanted to take back property that was rightfully his, he did not have the intent to commit a crime. Thus, his strongest defense to burglary is that he did not intend to commit a crime inside the house. (A) is incorrect. The breaking necessary for burglary requires some use of force to gain entry, but minimal force is sufficient. Opening a closed but unlocked door constitutes a breaking. Here, the man opened the unlocked back door, so his actions are sufficient for burglary. (B) is incorrect. Burglary requires that the defendant have the intent to commit a felony at the time of entering the dwelling, but it is not necessary that the felony be completed or even attempted. Here, the man intended to retrieve computer equipment, but he got scared and left the house without taking anything. The fact that he did not go through with his plan does not affect his culpability for burglary. (C) is incorrect. A structure is a dwelling if it is used with regularity for sleeping purposes. The occupants do not have to be present during the burglary, and the occupants' temporary absence does not deprive the structure of its character as a dwelling. Here, the employee and her family lived in the house and were away that night. Their temporary absence did not affect the house's status as a dwelling.
After her husband died in a hospital, a widow directed the hospital to send her husband's body to a funeral home for burial. The hospital negligently misidentified the husband's body and sent it to be cremated. When she was informed of the hospital's mistake, the widow suffered serious emotional distress. She has sued the hospital. Is the hospital likely to be held liable to the widow? Press Enter or Space to submit the answerANo, because the widow did not witness the cremation.BNo, because the widow was never in any danger of bodily harm.CYes, because hospitals are strictly liable if they do not properly dispose of corpses.DYes, because the negligent handling of the husband's body was especially likely to cause his widow serious emotional distress.
(D) is correct. In the usual action for negligent infliction of emotional distress, the plaintiff must be within the "zone of danger" and must suffer physical symptoms from the distress. However, the plaintiff may be able to recover without proving these requirements in special situations where the defendant's negligence creates a great likelihood of severe emotional distress, such as when the defendant mishandles the corpse of a family member of the plaintiff. Here, the hospital negligently sent the body of the widow's husband to be cremated, contrary to her wishes. This conduct caused her serious emotional distress, for which the hospital is likely to be liable. (A) is incorrect. While a bystander must witness a family member's injury to recover for negligent infliction of emotional distress under the bystander theory of recovery, the theory of recovery here is based on the obligation of the hospital to handle the deceased's body with due care. Failure to do so makes the hospital liable to the widow even though she did not witness the cremation. (B) is incorrect. As discussed above, the hospital may be liable to the widow for her emotional distress even though she was not in the "zone of danger" of bodily harm. (C) is incorrect. Hospitals are not strictly liable for the disposition of corpses. Rather, the hospital is liable for its negligence in mishandling the husband's corpse and causing the widow serious emotional distress.
A man was charged with first-degree murder. Two lawyers were appointed to represent him because the prosecution planned to seek the death penalty. On the first day of trial, the air-conditioning malfunctioned in the courtroom, so the judge directed that the selection of the jurors take place in his chambers. Because of the large number of potential jurors, the judge directed that only one lawyer for the prosecution and one for the defense participate in the jury selection process. The defendant remained in the courtroom during the questioning of the jurors. Once the jury was selected, the trial was postponed until the next day, when the air-conditioning was again working. Did the court's jury selection process violate the defendant's federal constitutional rights? Press Enter or Space to submit the answerANo, because a defendant's confrontation rights are limited to witnesses rather than jurors.BNo, because there was good cause to conduct jury selection outside the defendant's presence.CYes, because excluding the defendant undercut the presumption of innocence.DYes, because jury selection is a critical stage at which a defendant is entitled to be present.
(D) is correct. Jury selection is a critical stage of trial at which the defendant is entitled to be present. (A) may be true, but it is also off point. The problem here is not a confrontation right, but rather the right to participate in one's own case. (B) is incorrect because it does not present a proper legal standard. There is no "good cause" which would permit a judge to violate a defendant's constitutional guarantees regarding the right to a jury. (C) is incorrect because nothing in the facts present an issue regarding the presumption of innocence.
A woman was standing in the aisle of a subway car and put her purse on the seat next to her. A man approached the woman from behind and grabbed the purse off the seat. He then pushed the woman out of the way and ran out of the subway car while carrying the purse. The man was apprehended on the subway platform while in possession of the purse. In a jurisdiction that follows the common law with respect to criminal offenses, of what crime can the man properly be convicted? Press Enter or Space to submit the answerALarceny, because force was not used until after he took the purse.BLarceny, because he made no threat to use force.CRobbery, because he physically took the purse from the woman's presence.DRobbery, because he used force in leaving with the purse.
(D) is correct. Robbery is the taking of personal property of another from the other's person or presence by force or intimidation with the intent to permanently deprive her of it. The force or threats must be used either to gain possession of the property or to retain possession immediately after such possession has been accomplished. Here, the man grabbed the woman's purse off the seat and then pushed the woman out of the way as he ran out of the subway car. He used force to retain possession of the purse immediately after he grabbed it, which satisfies the force requirement. Therefore the man can properly be convicted of robbery. (A) is incorrect. As explained above, the force requirement for robbery is satisfied if force is used either to gain possession of the property or to retain possession immediately after such possession has been accomplished, as was the case here. (B) is incorrect. Robbery requires that the defendant use either force or threats to take the property. Threats are not required if the defendant uses force. (C) is incorrect. Although it is true that the man physically took the purse from the woman's presence, this fact does not distinguish robbery from larceny. Larceny is the taking and carrying away of personal property of another by trespass with the intent to permanently deprive the person of her interest in the property. Robbery is basically a form of larceny in which the taking is accomplished by force or threats of force. If the man had taken the purse without using any force against the woman, he would be guilty of larceny. The man is therefore guilty of robbery because he used force, not because he physically took the purse from the woman's presence.
A man decided to steal a valuable coin collection from a collector's house while the collector was away. Knowing that the house had an alarm system, the man contacted the pool cleaner who worked at the house twice a week. The man offered the pool cleaner part of the proceeds from selling the coin collection if she would disarm the alarm and leave a side door unlocked so that the man could enter the house. The pool cleaner pretended to agree but then contacted the police, who immediately arrested the man. In a jurisdiction that follows the common law and has adopted the bilateral requirement for conspiracy, what crime has the man committed? Press Enter or Space to submit the answerAAttempted burglary.BAttempted larceny.CConspiracy.DSolicitation.
(D) is correct. Solicitation consists of inciting, advising, or inducing another to commit a crime with the specific intent that the person solicited commit the crime. Here, the man asked the pool cleaner to disarm the alarm and leave a side door unlocked so that he could enter the house and steal the coin collection, and he offered her part of the proceeds from selling the collection. This was clearly an inducement for the cleaner to commit a crime, and the man had the specific intent that the cleaner commit the crime. He has therefore committed solicitation. (A) is incorrect. A criminal attempt is an act that, although done with the intention of committing a crime, falls short of completing the crime. An attempt therefore consists of two elements: (i) a specific intent to commit the crime, and (ii) an overt act in furtherance of that intent. The overt act must be an act beyond mere preparation for the offense. Burglary is the breaking and entering of the dwelling of another at nighttime with the intent to commit a felony therein. Here, the man did not take any action toward stealing the coin collection besides offering the pool cleaner money to help him with his plan. He did not commit any act that could be considered an act beyond mere preparation. In addition, it is unclear whether the crime contemplated would be burglary because it is not known how the man would enter the house and whether it would be at night. (B) is incorrect. Larceny is the taking and carrying away of the personal property of another by trespass with the intent to permanently deprive the person of his interest in the property. As explained above, attempt requires an overt act that goes beyond mere preparation. Again, the man did not do anything that could be considered an act beyond mere preparation in furtherance of an intent to commit larceny. (C) is incorrect. Conspiracy consists of an agreement between two or more persons, an intent to enter into an agreement, and an intent to achieve the objective of the agreement. Under the bilateral approach, at least two guilty minds are required for a conspiracy. If one person in a two-party agreement is only feigning agreement, the other party cannot be convicted of conspiracy. Here, the cleaner outwardly agreed to the man's plan, but she was only pretending and immediately contacted the police. Because she did not intend to enter into an agreement with the man and did not intend to help him steal the coin collection, she did not have a guilty mind. The man therefore cannot be convicted of conspiracy.
A federal statute extends federal minimum wage requirements to all dry cleaning stores. The statute contains express findings that, when combined, the wages received by dry cleaning workers have a substantial impact on the national economy and on the flow of goods and services in interstate commerce. These findings are supported by information presented to Congress during committee hearings on the legislation. A small dry cleaning store operates exclusively within a community in the center of a geographically large state. It has no customers from outside the state. It employs three workers, each of whom is paid less than the federal minimum wage. Must this dry cleaning store comply with the statute imposing the federal minimum wage requirements on all dry cleaning stores? Press Enter or Space to submit the answerANo, because the store does no business in interstate commerce.BNo, because the wages of the store's three workers do not have a substantial impact on interstate commerce.CYes, because the Commerce Clause vests Congress with plenary legislative authority over labor relations.DYes, because the wages paid by dry cleaning stores have a substantial impact on interstate commerce.
(D) is correct. The Constitution gives Congress the power to regulate interstate commerce. This power has been construed broadly to include the power to regulate channels of interstate commerce, such as roads and airways; instrumentalities of interstate commerce, such as trucks and trains; and economic or commercial activities - even ones that take place solely within one state - that in aggregate have a substantial effect on interstate commerce. Here, Congress has found that the wages received by dry cleaning workers have a substantial impact on the national economy. Moreover, payment of wages is an economic activity. Therefore, the law is within Congress's commerce power and the dry cleaning store must comply. (A) is incorrect because there is no requirement of interstate business, as stated above. Congress has the power to regulate intrastate economic activity that in aggregate substantially affects interstate commerce. (B) is incorrect for the same reason - Congress's power does not depend on whether these three workers' salaries have a substantial impact on interstate commerce, but rather on whether the impact of all dry cleaning workers in aggregate is substantial. (C) is incorrect because the Commerce Clause has no special rule for labor relations.
To improve the quality of rental housing within its boundaries, a city proposed an ordinance requiring all new and existing rental housing units to provide at least one full bathroom for each bedroom, plumbing and electrical hookups for a washer and dryer, and a covered parking space. A majority of the owners of existing rental housing in the city opposed the ordinance. They argued that it would dramatically decrease the number of low-income rental housing units because owners would be unable to raise rents enough to recoup the investment required to comply. Without denying these contentions, the city enacted the ordinance. A plaintiff who owns low-income rental housing has sued the city, claiming only that the ordinance is unconstitutional on its face. Which of the following best states the burden of persuasion in this action? Press Enter or Space to submit the answerAThe city must demonstrate that the ordinance is necessary to serve a compelling state interest, because it adversely affects the fundamental right of rental housing owners to use their property in the manner they deem most economically efficient.BThe city must demonstrate that the ordinance is necessary to serve a compelling state interest, because it will have a substantial and disproportionate negative impact on low-income persons.CThe plaintiff must demonstrate that the ordinance is not substantially related to an important state interest, because it requires some owners of rental housing to invest money that they will not be able to recoup from increased rents.DThe plaintiff must demonstrate that there is no rational relationship between the ordinance and any legitimate state interest, because the ordinance regulates economic activity of a type normally presumed to be within state regulatory authority.
(D) is correct. The claim underlying the challenge here must be that the ordinance discriminates against poor people. Under the Equal Protection Clause, the burden of persuasion depends on the basis of the classification and the nature of the right involved. If the basis of the classification is suspect or a fundamental right is involved, strict scrutiny applies and the government must prove its action is necessary to achieve a compelling interest. But if no fundamental right is involved, and the classification is not suspect or quasi-suspect (that is, based on sex or legitimacy), the person challenging the government action must prove that it is not rationally related to achieving a legitimate government interest. Here, the classification appears to affect people on the basis of wealth, which is not a suspect or quasi-suspect classification, and it affects the ability to obtain housing, which is not a fundamental right. Therefore, the rational basis test applies. (A) is incorrect. It correctly sets forth the burden when a fundamental right is involved, but the right to rent housing is not a fundamental right. (B) is incorrect for a similar reason - as discussed above, wealth is not a suspect classification. (C) is incorrect too. Although it correctly indicates that the plaintiff has the burden, it states the wrong test - when the challenger has the burden, the challenger must prove the state action is not rationally related to achieving any legitimate government interest.
A new gang member, hoping to impress the gang's leader, pointed a gun at a pedestrian and ordered her to give him her expensive watch, which she did. The gang member then tossed the watch to the gang leader, who was standing nearby. Although totally surprised by this act, the gang leader put the watch in his pocket. The pedestrian ran away. What crime did the gang leader commit? Press Enter or Space to submit the answerAAccessory after the fact to robbery.BAccomplice to robbery.CConspiracy to commit robbery.DReceiving stolen property.
(D) is correct. The crime of receipt of stolen property consists of receiving possession and control of stolen personal property known to have been obtained in a manner constituting a criminal offense by another person with the intent to permanently deprive the owner of his interest in the property. Here, the gang leader put the pedestrian's watch in his pocket after it was tossed to him by the gang member. The gang leader knew that the watch was obtained by a criminal offense because he was standing nearby when the gang member robbed the pedestrian. Finally, it can be inferred from the circumstances that the gang leader had the intent to permanently deprive the pedestrian of the watch. He has therefore committed the crime of receiving stolen property. (A) is incorrect. An accessory after the fact is one who receives, comforts, or assists another, knowing that he has committed a felony, in order to help the felon escape arrest, trial, or conviction. Here, the gang leader did not do anything to help the gang member escape arrest, trial, or conviction for robbing the pedestrian. He merely put the stolen watch in his pocket, which is insufficient for liability as an accessory after the fact. (B) is incorrect. An accomplice is one who, with the intent to assist the principal and the intent that the principal commit the crime, actually aids, counsels, or encourages the principal before or during the commission of the crime. The gang leader's conduct does not meet any of these requirements. He did not intend for the gang member to commit the robbery, and he did not aid or encourage the gang member before or during the robbery. The gang leader was totally surprised by the gang member's actions. He therefore is not an accomplice to the robbery. (C) is incorrect. Conspiracy requires an agreement between two or more persons, an intent to enter into an agreement, and an intent to achieve the objective of the agreement. As described above, the gang leader did not have the intent to commit robbery. There was no agreement between the gang leader and member; the leader was completely surprised by the member's actions. There was therefore no conspiracy to commit robbery.
A man entered a store with a gun, planning to rob the store. He was unfamiliar with guns and thought that the safety lock was on. When the store manager refused to give him any money, the man shook the gun at her to scare her into cooperating. The gun fired, killing the manager. A clerk ran in from the back stockroom. The man fired a shot into the air to scare the clerk. Unbeknownst to the man, the clerk had a heart condition, and the fright caused her to suffer a fatal heart attack. Can the man properly be convicted of felony murder? Press Enter or Space to submit the answerANo.BYes, but only for the death of the store manager.CYes, but only for the death of the clerk.DYes, for the deaths of both the manager and the clerk.
(D) is correct. The felony murder doctrine states that a killing-even an accidental one-committed during the course of a felony is murder. The death must be a foreseeable result of the commission of the felony. Here, the man was attempting to commit robbery, a felony, when he caused the death of the manager and the clerk. He shook the gun, causing it to fire and kill the manager. He later shot into the air to scare the clerk, which caused her to suffer a fatal heart attack. It is foreseeable that an armed robbery could result in deaths, especially when the defendant is shaking a gun and shooting it in the air. The clerk's heart condition does not relieve the man of liability because a victim's preexisting condition that makes her more susceptible to death does not break the chain of causation. Finally, it does not matter that the man did not intend to kill anyone; the felony murder doctrine applies to accidental deaths. He is therefore guilty of felony murder of both the manager and the clerk. (A) is incorrect. As explained above, the man can be convicted of felony murder of both the manager and the clerk. (B) is incorrect. The man is also guilty of felony murder of the clerk. (C) is incorrect. The man can also be convicted of felony murder of the manager.
A state generally provides funding for the medical care of its residents who cannot afford such care. State law, however, prohibits use of this state funding for surgery for any person who has resided in the state for less than one year, except in emergency situations. A woman moved to the state two months ago seeking permanent employment. Her physician recommends non-emergency surgery to treat a medical condition. The surgery would qualify for state funding if the woman had resided in the state for a year. The woman has sued to invalidate the state law that prohibits state funding of her surgery. Should the woman prevail in her action? Press Enter or Space to submit the answerANo, because the law reasonably conserves the state's limited resources.BNo, because the law reasonably prevents the expenditure of state funds on transient nonresidents.CYes, because the law burdens the woman's fundamental right to health care.DYes, because the law burdens the woman's fundamental right to travel.
(D) is correct. The right to travel is protected by the Fourteenth Amendment, and it includes the right of newly arrived citizens to enjoy the same privileges as are enjoyed by other citizens of the state. The law here gives a benefit to long-term residents (people who have been residents more than a year) and so infringes on the right to travel. (A) is incorrect. Because the right to travel is a fundamental right, it cannot be interfered with merely because it is reasonable to do so. Likely, the state would have to show that the interference is necessary to achieve a compelling interest. (B) is incorrect for the same reason that (A) is incorrect. (C) comes to the correct conclusion but for the wrong reason. There is no fundamental right to healthcare for women or men.
A contractor agreed to remodel a homeowner's garage for $5,000. Just before the parties signed the one-page written contract, the homeowner called to the contractor's attention the fact that the contract did not specify a time of completion. The parties orally agreed but did not specify in the contract that the contractor would complete the work in 60 days, and then they both signed the contract. The contract did not contain a merger clause. The contractor failed to finish the work in 60 days. The homeowner has sued the contractor for breach of contract. Is the court likely to admit evidence concerning the parties' oral agreement that the work would be completed in 60 days? Press Enter or Space to submit the answerANo, because the court must ascertain the meaning of the agreement from the terms of the written contract.BNo, because the oral agreement was merely part of the parties' negotiations.CYes, because the contract is ambiguous.DYes, because the time limit is an additional term that does not contradict the partially integrated written contract.
(D) is correct. The time limit is an additional term that does not contradict the partially integrated written contract, and evidence concerning it will be admissible. When the parties to a contract express their agreement in a writing with the intent that it embody the full and final expression of their bargain, the writing is an "integration," and under the parol evidence rule, admissibility of evidence seeking to vary its terms is limited. If the writing is only a partial integration, and not a complete embodiment of the parties' intentions, under the parol evidence rule, it cannot be contradicted, but it may be supplemented by proving up consistent additional terms. Here, the writing likely was only a partial integration - it was only one page, it did not include a merger clause (inclusion is strong evidence the parties intended the writing to be a full and final integration), and it did not specify a time of completion. The parties did not intentionally leave out the time of completion, it just wasn't brought to their attention until just before signing. At that point, the parties orally agreed to that supplementary term. Therefore, parol evidence would be allowed to supplement the terms of the partial integration. (A) is incorrect. If the writing here had been a fully integrated contract, the parol evidence rule would limit the terms of the contract to the four corners of the document. However, even in that case, a court is not limited to the document itself in interpreting its terms. For example, the court can look to usage of trade and course of dealing to interpret the meaning of a contract. (B) is incorrect. The agreement as to the time of completion was not part of the parties' negotiations. In fact, the parties failed to discuss the matter until it was called to their attention when they had finished negotiations and were about to sign the written contract. At that time, the parties orally agreed to the 60-day deadline, which is a consistent additional term. (C) is incorrect. The written contract was not ambiguous; rather, it was missing a time for completion.
A ceramics studio contracted with an artist to produce cups and saucers designed by the artist. The artist was an established designer of collectible ceramic dinnerware, and the studio did production work for many artists who created ceramic dinnerware. The price and quantity term of the contract read: "2,000 sets of the cups and saucers at $5 each, payable on delivery." The contract contained a merger clause. The studio produced the cups and saucers and delivered them along with a bill for $20,000 (4,000 pieces at $5 per piece). The artist refused to pay more than $10,000 (2,000 sets at $5 per set). At the trial of the studio's action against the artist for breach of contract, the studio introduced evidence of an established practice in the studio industry to price cup-and-saucer sets on a per-piece, not a per-set, basis. Is the studio's evidence admissible? Press Enter or Space to submit the answerANo, because such evidence would vary an unambiguous term of the contract.BNo, because the agreement was completely integrated.CYes, because evidence of trade usage is always admissible.DYes, because the usage of trade is offered to give meaning to the contract.
(D) is correct. This is a contract for the sale of goods (cups and saucers), so Article 2 of the UCC applies. The UCC provides that a written contract's terms may be explained or supplemented by evidence of course of performance, course of dealing, and usage of trade-regardless of whether the writing appears to be ambiguous. Thus, the studio's evidence will be admissible to give meaning to the terms of the contract. (A) is incorrect. As explained above, Article 2 allows terms to be explained or supplemented even if they do not appear to be ambiguous. (B) is incorrect. Even if the agreement is completely integrated under the parol evidence rule, evidence is admissible to define (as opposed to vary) the terms of the contract. (C) is incorrect. Evidence of trade usage is not always admissible (for example, it could not be used to vary the express terms of a fully integrated, final contract), but it can be used to explain or supplement terms.
A man was driving his new car along a dark road with the car's high-beam headlights on to illuminate the road. When he saw the headlights of another car appear in the distance, he reached to turn the high beams off. Instead of turning from high-beam to low-beam, the headlights on the car turned off completely. The man tried repeatedly to turn the lights on again but could not do so. He collided with the other car and suffered injuries. The man has brought an action against the manufacturer of the headlight controls in his car, the manufacturer of his car, and the retailer who sold him the car. If the man can establish that a defect in the controls caused the accident, from whom can he recover? Press Enter or Space to submit the answerAOnly the manufacturer of the car and the retailer of the car.BOnly the manufacturer of the car.COnly the manufacturer of the headlight controls and the manufacturer of the car.DAll three defendants.
(D) is correct. To establish a prima facie case in products liability based on strict liability in tort, the following elements must be proved: (1) the defendant is a commercial supplier; (2) the defendant produced or sold a defective product; (3) the defective product was the actual and proximate cause of the plaintiff's injury; and (4) the plaintiff suffered damages to person or property. A "commercial supplier" can include a manufacturer (including the manufacturer of a defective component part), retailer, assembler, or wholesaler. Here, all three defendants are commercial suppliers for purposes of the headlight controls if the man establishes that a defect in the controls caused the accident. (A) is incorrect. The manufacturer of the headlight controls is a commercial supplier of that product and would be liable if a defect in the controls caused the accident. (B) is incorrect. Both the manufacturer of the headlight controls and the retailer would also be liable if a defect in the controls caused the accident. (C) is incorrect. In a products liability action based on strict liability, a retailer may be liable for a manufacturing or design defect simply because it was a commercial supplier of a defective product-even if it had no opportunity to inspect the manufacturer's product before selling it.
A plaintiff sued an industrial facility in her neighborhood for injuries to her health caused by air pollution. At trial, the plaintiff was asked questions on direct examination about the days on which she had observed large amounts of dust in the air and how long the condition had lasted. She testified that she could not remember the specific times, but that she maintained a diary in which she had accurately recorded this information on a daily basis. When her attorney sought to refresh her recollection with her diary, she still could not remember. The plaintiff's attorney seeks to have the information in the diary admitted at trial. Is the information admissible? Press Enter or Space to submit the answerANo, because reviewing it did not refresh the plaintiff's recollection.BNo, unless it is offered by the defendant.CYes, and the plaintiff should be allowed the option of reading it into evidence or having the diary received as an exhibit.DYes, and the plaintiff should be allowed to read the diary into evidence.
(D) is correct. Where a witness states that she has insufficient recollection of an event to enable her to testify fully and accurately, even after she has consulted a memorandum or other record given to her on the stand, the record itself may be read into evidence if a proper foundation is laid for its admissibility. This is known as the recorded recollection exception to the hearsay rule. Here, all the foundational requirements are met-the plaintiff had personal knowledge of the facts when she made the diary; she made the diary herself and in a timely manner each day; she has vouched for its accuracy; and she currently has insufficient recollection to testify fully and accurately about the facts contained in the diary. Thus, the information in the diary may be read into evidence. (A) is incorrect. The fact that the witness cannot remember the facts even after reviewing the diary supports, rather than prevents, the information in the diary being admitted as a recorded recollection. Refreshing recollection is a separate concept from recorded recollection; a writing that is used to refresh recollection is not authenticated, is not in evidence, and may be used solely to refresh the witness's recollection. If the writing fails to refresh the witness's recollection, then it may qualify as a recorded recollection if the other foundational requirements are met. Thus, the fact that the plaintiff still cannot remember after reviewing the diary is crucial to its admissibility. (B) is incorrect. It is true that a recorded recollection may not be received by the jury as an exhibit unless offered by the adverse party; it can only be read into evidence otherwise. The rationale for this limitation is to avoid giving undue weight to the record, since it is being used as a substitute for in-court testimony and nothing more. However, this limitation is one of form, not admissibility, and here the information in the diary may be read into evidence. (C) is incorrect. The plaintiff does not have the option of having the diary received as an exhibit; it may be read into evidence only, unless an adverse party chooses to offer it.
A toy collector had purchased 10 antique toys over the last several years and had had them restored by an expert in toy restoration. On June 1, the collector sent the 11th antique toy to the expert with a signed note that read: "Here is another toy for you to restore. As with all prior jobs, I will pay $500 for the work, but no more." On June 4, after receipt of the collector's June 1 note and the toy, the expert began restoring the toy. On June 6, the collector unexpectedly died. On June 7, unaware of the collector's death, the expert sent the collector a note that stated that the restoration work had begun on June 4. The following day, the expert learned of the collector's death. Does a contract exist that binds the expert and the collector's estate? Press Enter or Space to submit the answerAYes, because the expert sent the June 7 note before learning of the collector's death.BYes, because the offer was accepted before the collector's death.CNo, because the collector died before the expert sent the June 7 note.DNo, because the offer lapsed when the collector died.
B) is correct. The offer was accepted before the collector's death. The collector's letter was an offer. It created a reasonable expectation in the expert that the collector was willing to enter into a contract on the basis of the offered terms. Unless an offer specifically provides that it may be accepted only through performance, it will be construed as an offer to enter into a bilateral contract and may be accepted either by a promise to perform or by the beginning of performance. Here, the expert began performance and, thus, accepted the offer when he began restoring the toy on June 4 - two days before the collector died on June 6th. (A) is incorrect. The collector's offer was accepted on June 4 when the expert began performance. Whether the expert knew of the collector's death on June 7 is not relevant. Indeed, if the expert had not already accepted the offer before the collector's death, the offer would have terminated on the collector's death by operation of law, even if the expert did not know of the death. (C) is incorrect. The June 7 note was not an acceptance. The acceptance had already occurred on June 4 when the expert began performance by beginning to restore the toy. (D) is incorrect. If either of the parties to a proposed contract dies prior to acceptance, the offer is terminated. Here the collector's offer had already been accepted at the time of the collector's death. Thus, there was no lapse.